Câu hỏi ôn tập môn Hoạt động kinh doanh Ngân hàng bằng tiếng Anh có đáp án

Câu hỏi ôn tập môn Hoạt động kinh doanh Ngân hàng bằng tiếng Anh có đáp án giúp sinh viên ôn luyện và nắm vững kiến thức môn học để đạt kết quả cao sau khi kết thúc học phần

lOMoARcPSD|36667950
1. Which market do banks participate in to buy and sell short-term securities? a. Currency market .
b. Capital market
c. Forex market
d. Gold market.
2. The service to pay money from customer's account to the address requested by the customer is:
a. Payment service using tax authorization
b. Payment service using periodic payment authorization
c. Treasury Services
d. Account management services and safes
3. Which of the following is not consistent with deposit management principles ?
a. The bank will provide information about the husband's savings deposit balance when requested by the
wife
b. The bank will not provide information about the husband's savings deposit balance at the request of
the wife (except for the couple who are co-owners).
c. The wife can use her own passbook as security for her husband's loan transaction at the bank.
d. All of the above sentences are correct
4. Banks are classified as financial intermediaries because...
a. Banks act as intermediaries in the markets
b. The main assets of a bank are financial assets
c. The operation of a bank consists of two stages: fund mobilization and fund use
d. Banks do not create their own claims, only transfer claims between the surplus and the deficit (s tin
thiếu ht)
5. Which statement most accurately reflects the concept of market risk?
a. Fluctuations (s biến động) in market price factors adversely (bt li) affect the value of assets of the
bank
b. Fluctuations in market price factors adversely affect the value of the bank's assets and liabilities
c. Fluctuations in market price factors adversely affect the value of assets or liabilities of the bank.
d. Fluctuations in market price factors adversely affect the value of assets and liabilities of the bank.
6. Distinguish (phân bit) between magnetic (t tính) and electronic cards in:
a. Origin of the card => debit card, credit card and prepaid card
b. The type of card reader suitable for it
c. Scope of card use
d. Immediate or deferred payment method
7. The minimum capital adequacy ratio (CAR coefficient) is calculated:
a. Own capital / Total assets at risk
b. Own capital/Total assets
c. Equity/Total Assets
d. Equity/Total assets at risk
8. Which of the following statements is not correct?
a. Risks in bank credit are inevitable (không th tránh khi) and cannot be completely eliminated (loi
b)
b. Full repayment of both principal and interest is the essence (bn cht) of bank credit
c. In the credit contract, there is no content that shows the customer's commitment to unconditionally
repay the bank when the debt is due.
lOMoARcPSD|36667950
d. Transactional assets in bank credit are diverse, which can be in the form of currency, real assets or
signatures.
9. Which items are not included in the “Liabilities” of the bank:
a. Common shares
b. Term Deposit
c. Repurchase agreement (tha thun mua li)
d. Bonds issued by banks
10.Payment by check will
a. Faster than paying by proxy
b. Slower than paying by ATM
c. All payment methods are the same
d. Not enough grounds (các căn cứ) to conclude the above ideas
11.In Vietnam today, the most influential factor on savings deposits of commercial banks is
a. Financial capacity (kh năng) of commercial banks
b. Service quality of NH
c. Interest rate of capital mobilization of commercial banks
d. Trading network of commercial banks
12.Which of the following statements correctly reflects the theory of portfolio management by economist
Markowitz?
a. Diversification in asset investment will help lower the risk of the whole portfolio than the risk of the
individual asset classes combined.
b. Diversification (đa dạng hóa) in asset investment will make the risk of the whole portfolio higher than
the risk of individual asset classes combined.
c. Specialization (chuyên môn hóa) in asset investments will help lower the risk of the entire portfolio
than the risk of the individual asset classes combined.
d. Specialization in asset investing will make the risk of the whole portfolio higher than the risk of the
individual asset classes combined.
13.Which of the following tools is considered an on-balance sheet tool to help banks reduce credit
portfolio risk?
a. Debt secularization (chng khoán hóa n)
b. Credit Risk Swap (hoán đổi ri ro tín dng)
c. Debt trading
d. None of the above tools
14.What is the minimum legal capital required by Basel I (1988)?
a. 5%
b. 6%
c. 7%
d. 8%
15.Which of the following valuable papers can be purchased and sold by commercial banks?
a. Treasury Bills
b. Government bonds
c. Saving
d. a and b
16.Which of the following is not true about Banker's Acceptance?
lOMoARcPSD|36667950
a. Not transferable
b. For a bank accepting a bill of exchange, agreeing to seal the acceptance is considered a form of
signature credit for the customer who is the issuer.
c. A bill of exchange issued by an enterprise
d. To be stamped “accepted” by the bank on the bill of exchange
17.Which of the following factors does not affect a bank's financial investment activities?
a. The development of financial markets
b. State investment incentives (ưu đãi) and support policies for hi-tech parks
c. Bank's own capital
d. Laws of the State
18.Which of the following services is not part of the treasury service of commercial banks?
a. Currency exchange at the request of the customer
b. Liquidation (thanh lý) of assets according to the wil l
c. Cash collection at customer's location
d. Transfer money overseas
19.Which of the following statements is not true about derivatives?
a. The birth of derivatives stems from (bt ngun t) the potential risk in contract transactions in the
economy
b. Derivatives are always born before an underlying contract transaction
c. Derivatives are a form of hedging (phòng nga) financial risks when performing economic contracts
d. Using derivatives will help customers prevent and reduce financial risks
20.Which of the following types of contracts does not belong to derivatives?
a. Futures contract
b. Swap contract
c. Credit agreement
d. Option contract
1. In order to improve the capital adequacy ratio (CAR), commercial banks need to take the following
measures:
a. Increase charter capital
b. Reduce risky assets
c. Increase in equity combined with a decrease in risky assets
d. Increase risky assets
2. Which of the following loan and customer evaluation methods is a qualitative (định tính) method?
a. Method of artificial nervous system
b. Judgment (phán đoán) method
c. Credit score method
d. All of the above are correct
3. When Commercial Bank A uses a ladder strategy (chiến lược bc thang/ phân tng) to invest in
securities in the financial market, if the maximum term for the portfolio is 10 years, the investment
amount will be divided into: a. 5 equal parts
b. 10 equal parts
c. 15 equal parts
d. 20 equal parts
4. Which of the following statements is not correct?
lOMoARcPSD|36667950
a. Commercial banks are directly engaged (kinh doanh) in insurance business
b. Commercial banks are allowed to contribute capital and buy shares of enterprises operating in the
insurance sector
c. Commercial banks are allowed to establish subsidiaries (công ty con) operating
in the field of collateral management
d. Commercial banks must establish subsidiaries to carry out insurance business activities
5. According to current regulations, a joint-venture commercial bank is a commercial bank established
in the form of a legal entity:
a. Limited company (including 1 member and 2 members or more)
b. Limited Liability Company and Joint Stock Company
c. Limited liability companies with two or more members and Joint stock companies
d. Limited liability company with two or more members
6. How is debt trading done?
a. Participating in a loan
b. Participating in time
c. Transfer
d. All of the above are correct
7. An enterprise A in Ho Chi Minh City wants to pay for goods to a supplier in a northern province with
a value of VND 10 billion. Enterprises should use vehicles
a. Travel Secrets
b. Bank card
c. Accreditative (y nhim)
d. commission
8. Which of the following statements is not true about derivatives?
a. The birth of derivatives stems from the potential risk in contract transactions in the economy
b. Derivatives are always born before an underlying contract transaction
c. Derivatives are a form of hedging financial risks when performing economic contracts
d. Using derivatives will help customers prevent and reduce financial risks
9. The trust matrix model (mô hình ma trn tin cy) is considered a marketbased pricing model
because:
a. The model of the machine complies with (phù hp vi) the laws of the market
b. The market price factor does not affect this model
c. This model uses the market probability distribution to determine default risk
d. This model not only determines the default risk event, but also determines the decrease in the market
value of the loan
10.The CAMPARI rule is...
a. Method of evaluating customers through factors: Cash (Cash), capacity (Ability), loan interest
(Margin), loan purpose (Purpose), loan amount (Amount), repayment (Repayment) and loan
guarantee
b. It is a quantitative assessment method
c. Both of these are true
d. Both of the above are wrong
11.Based on the card's technology, the bank's cards include:
a. Magnetic and chip cards
b. Magnetic and credit cards
c. Magnetic cards and ATM cards
d. Chip cards and ATM cards
lOMoARcPSD|36667950
12.Which of the following statements is not correct?
a. In essence(v bn cht), interest rates are a type of price and are influenced by supply and demand
b. Basic interest rate is the highest interest rate that banks apply to lend to
reputable (uy tín) customers
c. Base interest rate is the floor of lending interest rate
d. The base rate indicates (cho biết) the minimum income on the bank's loan amount
13.The main uses of payment cards are:
a. Cash withdrawals
b. Pay for goods and services
c. Loan
d. All of the above are true
14.Activities that are not financial investment activities of Vietnamese commercial banks include:
a. Contribute capital to buy shares of enterprises
b. Buying and holding shares of other credit institutions banks
c. Loans to business customers
d. Invest in the financial market
15.Corporate customers are not allowed to own any of the following deposit types
a. Saved money
b. Deposit payment
c. Term Deposit
d. Certificates of deposit
16.Banks are classified as financial intermediaries because...
a. Banks act as intermediaries in the markets
b. The main assets of a bank are financial assets
c. The operation of a bank consists of two stages: fund mobilization and fund use
d. Banks do not create their own claims, only transfer claims between the surplus and the deficit
17.The Bank purchases investment securities with fixed maturities and holds them to maturity in order
to:
a. Credit risk prevention
b. Hedging foreign exchange risk
c. Increase income from receiving interest at the end of the period
d. Price increases revenue from the difference between buying and selling prices
18.Which of the following services is not part of the treasury services of commercial banks?
a. Currency exchange at the request of the customer
b. Liquidation of assets according to the wil l
c. Cash collection at customer's location
d. Transfer money overseas
19.Which of the following is not the purpose of securities investment activities for commercial banks?
a. Provide liquidity to the bank when needed
b. Regulating the stock market
c. Diversify profitable activities
d. Minimize the risk of concentration
20.NHM mainly works by:
a. Equity contributed by shareholders
b. Capital mobilized in the economy, in society
c. Granted budget capital
lOMoARcPSD|36667950
d. Loans in the interbank market.
1. According to current regulations, domestic commercial banks are established and organized in the
form of:
a. Joint Stock Company or Limited Liability Company
b. State-owned company or 1-member LLC
c. JSC or Private Company
d. JSC or LLC 1 member
2. Choose the wrong sentence
a. Equity capital of commercial banks affects the reputation (uy tín) of the bank, the safety and
reputation of the bank is also an important factor in creating trust for customers, attracting a large
amount of deposits.
b. The strict control of the Central Bank such as interest rates, exchange rate policy, the regulation of the
required reserve ratio indirectly affects the capital mobilization of commercial banks.
c. Loans from other credit institutions and loans from the State Bank usually account for a large
proportion of the total business capital of commercial banks
d. The charter capital (vốn điều l) of the bank must be at least equal to the legal capital (vn pháp định)
3. Please choose the most correct statement
a. Current deposits usually have higher interest rates than demand deposits
b. Current deposits have less volatile (d thay đổi, biến động) balances than demand deposits
c. Paying deposit has a credit balance, it can also have a debit balance
d. Demand savings deposit with outstanding balance
4. According to ......., what types of banks include commercial banks, commercial banks, cooperative
banks?
a. Operational goals
b. Business method
c. Banking
d. Area of operation
5. Disadvantages of one-time payment via SBV
a. Slow payment speed
b. Only applicable to banks in the same system
c. Signature samples must be registered at the lead bank and other member banks d. High cost of use
6. To hedge against the risk of exchange rate decline at the time of receipt of foreign currency in the
future, the exporter can:
a. Exercising (thc hin) the right to buy foreign currency today
b. Exercising a foreign currency put option today
c. Sell foreign currency call options today
d. Exercising to sell foreign currency put options today
7. The bank has purchased a number of corporate bonds, in which the repurchase agreement was
agreed upon when issued. The issuer will buy back these bonds when
a. Market interest rates rise
b. Market interest rates fall
c. Market interest rates fall, bond prices increase by the redemption price and reach the agreed maturity
d. Bank in trouble
8. According to current regulations, which of the following statements is not true?
lOMoARcPSD|36667950
a. Credit institution is an enterprise that carries out one, several or all banking activities
b. Credit institution is a commercial bank that performs one, several or all banking activities
c. Commercial bank is a type of bank that can carry out all banking activities
d. Bank is a type of credit institution that can carry out banking activities.
9. Which of the following is an on-balance sheet transaction of a bank?
a. Buy corporate bonds
b. Advise
c. Investment Management
d. Insurance
10.Deposits in trust at ... not insured
a. Securities Company
b. NH
c. People's Credit Fund
d. Microfinance institutions
11.In which areas are the subsidiaries (công ty con) of commercial banks allowed to operate?
a. Secured asset management (qun lý tài sản đảm bo)
b. Forex trading (kinh doanh ngoi hi)
c. Factoring (bao thanh toán)
d. All of the above statements are correct
12.The mechanism (cơ chế) of action of credit risk swaps is similar to a. Credit insurance
b. Debt trading form
c. Securitization (chng khoán hóa)
d. All is incorrect
13.If the bank expects the spot rate in 3 months to be lower than the 3 month forward rate today,
how will the bank speculate on the price?
a. Buy foreign currency for a term of 3 months, at the end of the drought period, it will be sold for
immediate delivery
b. Selling foreign currency for a term of 3 months, at the end of the term will buy for immediate delivery
c. Buy foreign currency with term of 3 months, at the end of term will buy spot
d. Selling foreign currency for a term of 3 months, at the end of the term will sell for immediate delivery
14.Which of the following statements is not related to concentration risk (ri ro tp trung)?
a. Ratio of using short-term sources for medium and long-term loans does not exceed 30%
b. For 1 customer, no more than 15% of the bank's own capital
c. The ratio of outstanding loans to non-production loans does not exceed 16% of total outstanding loans
d. The proportion of loans for securities business does not exceed 20% of the bank's charter capital
15.Which of the following is an indirect credit transaction?
a. Loan under overdraft (thu chi) limit
b. Discount negotiable
c. Financial leasing
d. Lending each time (cho vay tng ln)
16.Which asset class is highly liquid and profitable?
a. Gold, foreign currency
b. Corporate bonds
c. Business capital contribution
d. Government bonds
lOMoARcPSD|36667950
17.What measures should commercial banks not apply when short-term liquidity surplus?
a. Interbank deposit lending
b. Borrow on interbank
c. Invest in short-term valuable papers
d. Both b and c are correct
18.Which of the following is a qualitative assessment method for customers and loans?
a. Method of artificial nervous system
b. Judgment method
c. Credit score method
d. All of the above are correct
19.Attracting deposits by technical means including
a. Arrangement of ATMs at colleges and universities
b. Issuing multi-purpose cards, diversifying customers, providing white check services for customers
c. Issuing multi-purpose cards, giving raincoats to customers, saving money, diversifying deposit
methods
d. Issuing ATM cards at colleges and universities, providing blank check services to customers, increasing
the number of bank staff.
20.The Credit Scoring system is used to
a. Credit officer management
b. Customer analysis in deposit transactions
c. Improve the quality of payment by credit card
d. Slap all of the above ideas wrong.
1. Similarities in check payment method and payment order payment method
a. Payment certificate issued by the debtor
b. Chances of encountering fake vouchers
c. Is transferred directly in the delivery
d. All right
2. When we say securities with high liquidity, we mean
a. Such securities are readily convertible (có th chuyển đổi) to cash
b. Such securities are difficult to resell (bán li) in the market
c. Difficult to convert such securities into cash
d. Stocks with high prices
3. Which source of capital is the capital source to pay interest of commercial banks?
a. Deposits and loans to the Treasury, the State Bank and other credit institutions b. Other Receivables
c. Deposits from customers and issuance of valuable papers
d. Both a and c
4. Distinguish between electronic cards and electromagnetic cards in:
a. Origin of the card
b. The type of card reader suitable for it
c. Scope of card use
d. Immediate or deferred payment method
5. The credit portfolio risk measurement model helps banks:
a. Find a group of customers with absolutely no risk
b. Assess the working ability of credit officers
c. Increase credit scale without fear of any risk
d. All of the above are incorrect
lOMoARcPSD|36667950
6. What is the maximum capital contribution ratio of a bank compared to the value of its own capital
and reserves.
a. 30%
b. 40%
c. 50%
d. 60%
7. Which item does not belong to the profitable assets of commercial banks.
a. Term deposits of domestic and foreign credit institutions
b. Term deposits with economic and residential institutions
c. Investment in securities
d. Residential loan (cho vay mua nhà)
8. Money management service on account.........
a. Coupled with deposits, it should be an on-balance sheet transaction
b. Usually only applied to individual customers with long-term deposits
c. Usually applied to customers with large deposit balances
d. It is an unprofitable business because it is off-balance sheet
9. The KMV model is often used to.
a. Invest in gold
b. Stock investment
c. Invest in bonds
d. Foreign currency investment
10.A bill of exchange accepted by a bank is considered a highly secure instrument because:
a. The bank is first liable, having to pay the entire debt it secures to the beneficiary.
b. Issued and guaranteed by the government
c. Low interest rates
d. Absolutely no risk
11.Which of the following is not included when calculating loan interest?
a. Level of default risk
b. Tenor (k hn)
c. C cost of deposit mobilization
d. Cost rent
12.In the entrusting business:
a. The entrusting party transfers the property rights to the trustee under the contract
b. The trustee transfers the entire and permanent property rights to the trustee
c. The trustee operates for the benefit of the beneficiary
d. Both a and c are correct
13.Which of the following statements is correct?
a. Due to the phenomenon (hiện tượng) of information asymmetry (bt cân xng), the bank has the
ability to accept a bad customer at the same time as rejecting a good customer.
b. Due to the phenomenon of information asymmetry, the bank is able to accept a good customer while
at the same time rejecting a bad customer.
c. Both of the above are correct
d. Both of the above are wrong
14.Corporate customers may not own any of the following:
a. Saved money
b. Deposit payment
c. Term deposit
lOMoARcPSD|36667950
d. Certificates of deposit
15. In order to improve the capital adequacy ratio (CAR) h s an toàn vn,commercial banks need
to take measures. a. Increase charter capital
b. Reduce risk assets
c. Increase in equity combined with (kết hp vi) a decrease in risky assets d. Increase risky assets
16. Transactional and non-trading deposits are distinguished from each other oqr
a. Customers send money
b. Deposit term
c. Purpose of sending money
d. Sentences b and c are correct
17. Property guardian (giám h) service
a. As guardian of the client's previous activity
b. Asset management in case the heir is a minor (chưa thành niên)
c. Manage all customer needs
d. All three factors above
18. Interest rate risk occurs when
a. Fluctuating market interest rates affect the bank's net interest income and capital value.
b . Fluctuating market interest rates adversely affect a bank's net interest income and/or capital value
c. Rising market interest rates affect the bank's net interest income and capitalvalue
d. Falling market interest rates affect the bank's net interest income and capitalvalue
19. According to current regulations, a joint-venture commercial bank is acommercial bank
established with the form of a legal entity:
a. Limited Liability Company (including 1TV AND 2TV or more)
b. Company Limited and Joint Stock Company
c. Limited liability companies with 2 or more members and JSCs
d. Limited liability company with 2 or more members
20. Which of the following factors is not necessary to be negotiated (được đàm phán) in the credit
risk swap contract? a. Reference property
b. Credit risk events
c. Payment methods
d. Exchange rate changes in the market
1. In Vietnam, the operation of a financial institution is similar to that of an investment bank:
a. Insurance company
b. Financial leasing companies
c. Commercial Bank
d. Securities Company
2. Which of the following methods represents a bank's credit operations? a. Transfers
b. Thanks to the smooth collection
c. Thanks for collecting vouchers
d. Issuance of L/C
3. Which of the following operations cannot be grouped with the rest:
a. Personal loan
b. Bank guarantee
c. Other bank loans
lOMoARcPSD|36667950
d. Discount valuable vouchers
4. Arrange the following sentences in the correct order of payment:
1. Buyer and seller have a relationship to buy and sell goods and/or services with each other
2. Buyer's bank pays the seller through the seller's bank
3. The buyer writes a payment order to ask his bank to pay for the purchase to the seller
4. The seller's bank notifies (thông báo) the seller that the buyer has paid a. 1, 2, 3, 4
b. 1, 3, 4, 2
c. 1, 3, 2, 4
d. 1, 2, 4, 3
5. The objective of the trust matrix model is to:
a. Building a business strategy for the bank
b. Completing the bank's credit policy
c. Use calculation methods to measure the minimum risk exposure of each loan as well as the entire
portfolio
d. Assess (đánh giá) the quality of the bank's investment
6. Which of the following statements is not true about the operation of the NHS? a. Provide financial
advice
b. Issuing certificates of deposit, promissory notes, bills, and bonds to raise capital
c. Using mobilized capital to contribute capital, buy shares
d. Open an account to pay at another credit institution
7. Bancassurance is:
a. The combination of money transfer and insurance business
b. Combination of deposit and insurance operations
c. Combination of credit and insurance operations
d. Combination of banking and insurance
8. What activities does the commercial bank directly perform:
a. Underwriting of securities issuance
b. Insurance Business
c. Financial leasing
d. All is incorrect
9. Which sentence is not the same between payment by payment order and collection order?
a. There are 2 forms of payment through payment intermediaries
b. Orders made by the account holder must be made on pre-printed forms provided by the payment
service provider
c. Is a form of payment that the account holder makes an order, authorizes (y nhim) payment with
payment service providers to make payment.
d. Only used to pay different values, different periodicity (chu k)
10. Question: Credit institutions contribute capital or buy shares of enterprises by:
a. Charter capital and reserve fund
b. Charter capital and reserve fund, mobilized capital
c. Charter capital and mobilized capital
d. Charter capital and reserve fund, and borrowed capital
11. The basis for determining the floating interest rate for loans is:
a. Basic interest rate announced by the State Bank plus expected inflation
lOMoARcPSD|36667950
b. Savings deposit interest rate plus a certain margin
c. Real interest rate plus announced interest rate
d. All is incorrect
12. The main assets on a commercial bank's balance sheet are:
a. Deposits
b. Workhouse and office equipment
c. Loans to customers
d. Guarantees
13. Which of the following statements is false?
a. Anonymous sec (séc n danh) is a type of sec that does not include the beneficiary's name; and
anonymous checks are only transferable by endorsement
b. Identifier Sec is a type of sec specifying the beneficiary's name; sec identifier is not transferable
without endorsement
c. Transfer Sec is not transferable and cannot be cashed
d. Cash Sec is paid by the paying bank in cash
14. Which of the following tools cannot be used to reduce credit portfolio risk?
a. Credit risk swaps
b. Option contract
c. Futures contract
d. b and c are both correct
15. How is debt trading done?
a. Participating in a loan
b. Participating deadlines
c. Transfer
d. All right
16. Banking operations related to commercial paper include:
a. Collect commercial paper
b. Discount negotiable
c. Storm received commercial paper
d. All right
17. The meaning of securities investment activities for commercial banks is to:
a. Provide liquidity when needed
b. Diversify profitable activities
c. Increase profits
d. The sentences above are all correct .
18. The form of payment is made between commercial banks of different systems and locations, but each
commercial bank that opens a deposit account at the other bank is
a. Inter-bank payment
b. Payment via SBV
c. Clearing
d. Payment via agent bank relationship
19. If the customer's equity level in the plan is too low, this is related to... a. Choice risk
b. Business risk
c. Guaranteed risk
lOMoARcPSD|36667950
d. Intrinsic risk
20. The minimum capital adequacy ratio (CAR) reflects the relationship between
a. Own capital and risky assets of commercial banks
b. Own capital and converted risk assets of commercial banks
c. Own capital and assets of commercial banks
d. Own capital and credit balance of commercial banks.
Which asset class is highly liquid and profitable:
a. Gold, foreign currency;
b. Corporate bonds;
c. Contributing business capital;
d. Government bonds;
1. Streamlining the portfolio management process:
1. Consulting on investment types
2. Define Expectations
3. Sign investment portfolio management contract
4. Build the optimal investment structure.
5. Provide information
6. End of contract
a. 1,2,3,4,5,6;
b. 3,5,1,2,4,6;
c. 5,3,1,2,4,6;
d. 1,5,3,2,4,6;
2. Significance of securities investment activities for commercial banks:
a. Customer service development;
b. Brand enhancement;
c. Provide liquidity when needed;
d. Minimizing investment risks;
3. According to current regulations, for the performance of securities brokerage activities, commercial
banks…
a. Absolutely not allowed ;
b. Unlimited exercise is allowed ;
c. Direct execution is not allowed ;
d. Permitted through the Stock Exchange ;
4. Which of the following statements is not true about derivatives?
a. The advent of derivatives stems from the potential risks in contract transactions in the economy ;
b. Derivatives are always born before an underlying contract transaction ;
c. Derivatives are a type of financial hedging when performing economic contracts ;
d. Using derivatives will help customers prevent and reduce financial risks ;
5. Which of the following types of contracts does not belong to derivatives? a. Futures contract ;
b. Swap contract ;
c. Credit contract ;
d. Option contract ;
6. Which of the following is not related to portfolio management?
a. Define expectations ;
b. Building the optimal (tối ưu) investment structure ;
c. Consulting on investment types ;
lOMoARcPSD|36667950
d. Securities investment ;
7. Credit institutions contribute capital to purchase shares of enterprises by:
a. Charter capital and reserve fund;
b. Charter capital and reserve fund, mobilized capital;
c. Charter capital and mobilized capital;
d. Charter capital and reserve fund, borrowed capital;
8. Which function is not of securities investment activities:
a. Create geographical diversification;
b. Strengthen the capacity of enterprises to realize business opportunities;
c. Mitigating (gim thiu) the impact of taxes;
d. Balance credit risk ;
9. Which is not the meaning of securities investment activities for commercial banks:
a. Provide liquidity when needed;
b. Acquisition of businesses;
c. Diversify profitable activities;
d. Reduce the risk of concentration;
10. When commercial bank A uses a ladder strategy to invest in securities in the financial market, if the
maximum maturity for the portfolio is 10 years, the investment amount will be divided into:
a. 5 equal parts ;
b. 10 equal parts ;
c. 15 equal parts ;
d. 20 equal parts ;
11.Which of the following statements is not true about the ladder strategy in financial investment
activities of banks?
a. The bank is easy to manage, just allocate (phân b) investment evenly for the terms and regularly
check the due accounts for reinvestment ;
b. Limit fluctuations in the portfolio's income, by maintaining an average return, and avoiding major loss
events ;
c. Banks need to forecast future interest rates ;
d. All of the above are correct ;
12.Choose the best statement about the term division strategy (A spit-maturity approach)?
a. Fully invest in short-term securities ;
b. Fully invest in long-term securities ;
c. Investing in short-term and long-term securities ;
d. The term division strategy includes all three sub-strategies mentioned above ;
13. Banks use Barbell strategy in stock investment, ire:
a. Portfolio with only short-term securities ;
b. The portfolio contains only long-term securities ;
c. Portfolio with a combination of short-term and long-term securities ;
d. All of the above are correct ;
14.Which of the following statements is not true about the term division strategy?
a. With this strategy, the bank can get larger profits than the ladder strategy ;
b. This strategy is considered an active investment strategy ;
c. Banks do not need to predict the direction of interest rate movements in theeconomy ;
d. The bank may incur a loss if the bank's predictions are inaccurate ;
lOMoARcPSD|36667950
15.Which of the following statements is not true about derivatives?
a. The advent of derivatives stems from the potential risks in contract transactions in the economy ;
b. Derivatives are always born before an underlying contract transaction ;
c. Derivatives are a type of financial hedging when performing economic contracts ;
d. Using derivatives will help customers prevent and reduce financial risks ;
16. Which of the following types of contracts does not belong to derivatives? a. Futures contract ;
b. Swap contract ;
c. Credit contract ;
d. Option contract ;
17. Which of the following is not related to portfolio management?
a. Define expectations ;
b. Building the optimal investment structure ;
c. Consulting on investment types ;
d. Securities investment ;
18.The KMV model is often used to:
a. Gold investment;
b. Stock investment;
c. Bond investment;
d. Foreign currency investment;
19.Secularization:
a. Is the issuance of securities on the basis of the value of playable that a bank already owns;
b. Is the issuance of bonds on the basis of the value of playable that a bank already owns;
c. Is the issuance of securities on the basis of the value of receivables that a bank already owns;
d. Is the issue of shares on the basis of the value of the receivables that a bank already owns;
20.The bank has purchased a number of corporate bonds, in which the repurchase agreement was agreed
upon when issued. The issuer will redeem these bonds when:
a. Market interest rates increase;
b. Market interest rates fall;
c. The market interest rate decreases, the bond market price increases by the
redemption price and the agreed maturity;
d. Banks in trouble;
21.Which of the following is true?
a. Treasury bills are government debt certificates issued by the State Bank;
b. Treasury bills are government debt certificates issued by the State Treasury;
c. Treasury bills are not transferable on the money market;
d. All of the above are correct;
22.The Bank purchases investment securities with fixed maturities and holds them to maturity in order to:
a. Credit risk prevention ;
b. Hedging foreign exchange risk ;
c. Increase income from receiving interest at the end of the period ;
d. Increase revenue from the difference between buying and selling prices ;
23.If a commercial bank wants to distribute (phân phi li) securities investment fund certificates:
a. An investment fund must be established under ;
b. Must establish a securities brokerage company ;
c. Must acquire securities portfolio implementation (thc hin) company ;
d. All of the above are correct ;
lOMoARcPSD|36667950
24.Which of the following statements is most correct?
a. Government bonds have the lowest credit risk and the highest return ;
b. Government bonds have the lowest credit risk but at the same time have low profitability ;
c. Corporate bonds also have low credit risk ;
d. All of the above are correct ;
25.Commercial banks are allowed to contribute capital or purchase shares of enterprises operating in
which of the following fields?
a. Insurance ;
b. Securities ;
c. Credit information ;
d. All of the above are correct ;
26.What kind of valuable papers are not issued by the enterprise:
a. Commercial deed (chng thư thương mại)
b. Bill of exchange accepted by the bank ;
c. Certificate of Deposit ;
d. Corporate bonds ;
27.A bank's portfolio should be designed so that:
a. Guarantee the lowest cost ;
b. Ensure profits at a certain level of expectations but risks must be at a minimum ;
c. Highest return and acceptable risk ;
d. b, c are both correct ;
28.The bank used mobilized sources to buy bonds with a fixed rate of return, while deposit interest rates
increased according to the market. How will this affect the bank?
a. Risk of default ;
b. Interest rate risk ;
c. Credit risk ;
d. Completely unaffected ;
29.Which of the following is not the purpose of securities investment activities for commercial banks?
a. Provide liquidity to the bank when needed ;
b. Regulating (điều tiết) the stock market ;
c. Diversify profitable activities ;
d. Reduce the risk of concentration ;
30.Banks invest in fixed-yield securities such as government bonds. How will an increase in market interest
rates affect the market price of previously issued debt securities?
a. Increases the market price of debt securities ;
b. Decreasing the market price of debt securities ;
c. Does not change the market price of the debt securities ;
d. All of the above are wrong ;
31. Banks invest in fixed-yield securities such as government bonds. How will an increase in market
interest rates affect the market price of previously issued debt securities?
a. Increases the market price of debt securities ;
b. Decreasing the market price of debt securities ;
c. Does not change the market price of the debt securities ;
d. All of the above are wrong ;
32. Which of the following factors does not affect a bank's financial investment activities?
a. The development of financial markets;
lOMoARcPSD|36667950
b. The State's preferential policies and investment support for hi-tech parks;
c. The bank's own capital;
d. Laws of the State;
33. The bank's portfolio will focus on securities such as government bonds, Treasury bills, because:
a. These types of securities are not transferable on the market ;
b. These securities have low credit risk and are easily transferable in the market to supplement liquidity
when needed ;
c. These types of securities have higher profitability than others ;
d. All of the above are correct ;
34. Financial investment activities help banks:
a. Increase the balance of term between capital sources and use of capital ;
b. Limit interest rate risk ;
c. Dispersing credit risks ;
d. All of the above are correct ;
35. In some cases, a bank's portfolio is not for profit, but for:
a. Use idle money;
b. Create liquidity;
c. Increased riskiness;
d. Increasing the bank's competitiveness;
36. Which of the following investment securities is the least safe? a. Treasury Bonds;
b. Treasury Bills;
c. Banker's Acceptance of Bill of Exchange;
d. Corporate bonds;
37. When investing in securities, credit risk occurs when:
a. Market interest rates fluctuate;
b. The commercial bank has not used up its credit limit;
c. Investors massively sold securities;
d. The issuer of securities is unable to pay the principal and interest;
38. Why do investors prefer issuer callable bonds?
a. Stable income;
b. Not affected by fluctuations in the market;
c. These securities have higher yields than others;
d. High degree of safety;
39. REPO transactions include:
a.Selling securities
b. Buyback of securities
c.Only b is correct skin and
b
40. When you say securities are highly liquid, what do you mean?
a. Such securities are readily convertible to cash;
b. Such securities are difficult to resold in the market;
c. Difficulty converting such securities into cash;
d. Securities with high selling price;
41. Vietnamese commercial banks performing securities underwriting operations:
a. It is not necessary to have the approval of the State Bank;
b. The Law on Credit Institutions 2010 does not allow commercial banks to carry outsecurities
underwriting operations;
c. Must establish a securities company (subsidiary of the Bank);
lOMoARcPSD|36667950
d. Must be done at the Stock Exchange;
42. The influence of political and economic events causing a sharp drop in stock prices on the stock
market belongs to:
a. Interest rate risk;
b. Credit risk;
c. Market risk;
d. Risk of acquisition;
43. Which market do banks participate in to buy and sell short-term securities?
a. Money market ;
b. Capital market ;
c. Foreign exchange market ;
d. Gold market ;
44. Activities that are not financial investment activities of Vietnamese commercial banks include:
a. Contribute capital to buy shares of enterprises
b. Buying and holding stocks of other credit institutions/banks
c. Loans to business customers
d. Invest in the financial market
45. Which of the following valuable papers can be purchased and sold by commercial banks?
a. Treasury bills ;
b. Government bonds ;
c. Savings book ;
d. a and b ;
46. What kind of valuable paper is considered a certificate of debt?
a. Commercial paper ;
b. Company shares ;
c. Dividends ;
d. All of the above are correct ;
47. Investment activities help banks improve liquidity by:
a. Convert financial assets into cash according to the fourth priority: Highly liquidassets will be sold later
;
b. Convert financial assets into cash in order of priority: Assets with low liquidity aresold first ;
c. Convert financial assets into cash in order of priority: Liquid assets are sold first ; d. All of the above
are correct ;
48. Treasury bills are issued to:
a. Raising capital for the company ;
b. Serving the company's investment and development needs ;
c. To make up for the shortfall of the State Budget ;
d. Increase spending sources for the company ;
49. Which of the following is not true about corporate bonds?
a. As a type of debt security ;
b. Issued to finance local infrastructure projects ;
c. Confirm the debt repayment obligations of the issuer towards bondholders ;
d. Is one of the valuable papers that the Law allows banks to participate in themarket ;
50. What are the basic similarities between treasury bills and treasury bonds?
a. Release period ;
b. Release purpose ;
c. Issuing organization ;
d. All three of the above comments are correct ;
51. The joint venture capital of commercial banks is deducted from?
a. Charter capital of the bank ;
b. Deposits mobilized from customers ;
lOMoARcPSD|36667950
c. Bank's reserve fund ;
d. a and c ;
52. What is the method of securities underwriting in which the underwriter will commit to underwriting
all of the issued securities whether or not they are fully distributed?
a. Guarantee with commitment to try the best ;
b. Guarantee with firm commitment ;
c. Guarantee according to the minimum - maximum method ;
d. All of the above are wrong ;
53. What is not true about treasury bills?
a. Government debt certificates issued by the State Treasury ;
b. Issued by a private company ;
c. Instruments may carry no interest or low interest rates ;
d. Issued to make up for the deficit of the State Budget and realize the objectives of the monetary policy
;
54. Commercial banks participate in the money market in which of the following forms?
a. Participating in Treasury bill bidding
b. Buying and selling negotiable instruments, Government bonds
c. Buy State Bank bills
d. All of the above are correct;
55. A bill of exchange accepted by a bank is considered a highly secure instrument because:
a. The bank is first liable, having to pay the entire debt it secures to the beneficiary.
b. Issued and guaranteed by the Government
c. Low interest rates
d. Absolutely no risk
Part 2
1. Which method shows the bank's credit operations?
a. Transfers;
b. Thanks to the smooth collector;
c. Thanks to the collection of documents;
d. Issuing letters of credit;
2. What are the current forms of payment via bank?
a) Clearing, payment under contract, UNC, check;
b) Plan payment, transfer payment, cash payment;
c) Payment through multiple banks, payment through one bank;
d) Checks, payment orders, collection orders, letters of credit, card payments.
3. The payment range of Wire Transfer Checks is:
a) Within the national economy: between businesses, between businesses and individuals, between
individuals and businesses;
b) Within the scope of commercial banks in the same system, other commercial banks in the system,
commercial banks with the State bank, the State bank with commercial banks and bank branches with
each other;
c) Among customers who have opened accounts at the same or different banks but these banks
participate in clearing in the province or city;
d) Within the range between customers who open accounts at the same bank.
4. Which of the following accounts are not used in the Interbank Payment System:
a) Payment deposit accounts of members;
b) Clearing account;
c) Collection and payment accounts;
d) Customer's savings account.
lOMoARcPSD|36667950
5. Places where checks can be paid:
a) At the place of business of the drawee;
b) At the place indicated on the check;
c) At any credit institution, the bank is allowed to issue checks;
d) All 3 statements are true.
6. Which of the following is not a form of payment between banks:
a) Payment via State bank;
b) Deferred payment;
c) Clearing;
d) Payment through correspondent banking relationship.
7. : Which is not the advantage of clearing between banks:
a) Fast, can handle multiple payment transactions at the same time;
b) Creating conditions for banks to effectively manage capital and harmonize capital;
c) Property safety;
d) Clearly show the receivable and payable amount of each bank participating in clearing.
8. : If the amount written in words and numbers on the face of the promissory note do not match, the
payment will be based on which amount:
a) Amount in words;
b) Amount in number;
c) The smaller of the 2 numbers;
d) That bill of exchange has no value.
9. Delays in issuing a spending authorization can harm:
a) Buyer;
b) Seller;
c) Buyer and seller;
d) The bank provides payment services to buyers.
10.Which of the following is not true about payment methods in the form of collection orders:
a) Can be used in case the two parties buy - sell have mutual trust, have agreed on payment conditions;
b) Due to the beneficiary making the payment process may be delayed;
c) As established by the beneficiary, the payment process does not occur without delay;
d) The beneficiary's bank notifies the buyer through the buyer's bank.
11.: In what cases can the payment method by collection order be used?
a) In case two service providers do not have mutual trust;
b) In cases where the two parties buying and selling goods or providing services do not have mutual
trust or in the case of payment for services provided, there are accurate measuring devices such as
electricity, water, and telephone;
c) In case the two parties purchase and sell goods with mutual trust;
d) In cases where two parties purchase and sell goods or provide services with mutual trust or in the
case of payment for services provided, there are accurate measuring devices such as electricity,
water, and telephone;
12.Which of the following statements is not true about the characteristics of the clearing method between
banks?
a) Wide payment range;
b) The member banks must open a deposit account at the lead bank;
c) Can process multiple payment transactions at the same time;
d) The lead bank is the State Bank or another banking unit if there is no branch of the State Bank in the
locality.
13. Credit cards have the following advantages:
a. Low transaction fees and narrow operating range.
lOMoARcPSD|36667950
b. Popular for car buying and selling activities.
c. Use payment as an alternative to cash but not a substitute for checks.
d. The card can be used to withdraw cash from a bank branch, correspondent bank or ATM or transfer to
another bank's account;
14. which of the following is incorrect. When paying a check the customer must:
a. Have an account at that bank;
b. Payment can only be made within the same bank system;
c. Have a serial number in the bank's serial number;
d. Write the amount on the check to be less than the account balance;
15. Which is not the same thing between the two check and card payment methods?
a. All issued by the bank;
b. As a form of non-cash payment;
c. The payee can receive cash or wire transfer;
d. It is a form of payment through a bank intermediary.
16. What is the difference between UNC and UNT?
a. The beneficiary and the payer must have an account at the bank;
b. All created by the customer;
c. Service fee for money transfer paid by the beneficiary or the payer;
d. Make payments on UNC, UHT forms of NH.
17. Which form of payment is the supplier of goods and services more
guaranteed to pay? a. Sec ;
b. Authorization for collection;
c. Accreditative;
d. Letter of credit.
18. Credit card features:
a. Any customer can have a credit card;
b. When using a credit card, the debt balance decreases gradually;
c. The cardholder needs to have a balance on his payment account at the card issuing bank;
d. A credit card has the same features as a debit card.
19. Please choose False:
Members participating in the NIEPS must be payment service providers and comply with the following
conditions:
a. Must have a payment deposit account at the Exchange;
b. Must maintain the balance of the payment deposit account to secure the execution of
Payment Orders and clearing through the NIEPS;
c. Must establish, maintain and manage net debt limit in case of low value payment service;
d. With the consent of the direct management member.
20. Why distinguish between the presentation period and the payment
validity period:
a. To encourage the beneficiary to present the check promptly;
b. To encourage the use of check payments;
c. Extend the payment time, increase the validity period for the check, in case of unexpected
cases;
d. Grounds for considering the drawer's violation of payment discipline;
21. : What types of checks require the beneficiary to have a bank
account
a. Crossed checks, wire transfers;
b. Bank transfer checks, travelers checks;
c. Wire transfer check, order check;
d. Traveler's check, slash check.
lOMoARcPSD|36667950
22. Choose the correct answer:
a. The payment order must be made and signed by the payer and only based on that order for
the bank to pay money to the beneficiary;
b. The payment conditions of the payment order are that the beneficiary and the paying unit
must have a bank account;
c. Debit cards have functions such as: direct payment, bank loan for payment, automatic
withdrawal;
d. The credit card has functions such as: automatic withdrawal, direct payment, can carry the
balance over many consecutive months.
23. Choose the correct answer:
a. ATM card used for payment and withdrawal;
b. Debit cards are for withdrawals only;
c. Credit card for payment;
d. Payment card used to withdraw money.
24. Interbank payments are made between which of the following
banks?
a. Agribank district 9 branch and Vietinbank Binh Duong branch;
b. Vietcombank branch in district 9 and Vietinbank branch in district 9;
c. Agribank district 9 branch and Agribank Binh Duong branch;
d. Vietcombank branch in district 9 and Agribank branch in Binh Duong district;25. Which of the
following statements is correct?
a. To send money through another bank, the depositor must have an account at the bank;
b. To organize payment, 2 banks need each other's deposit accounts;
c. The main form of payment of the two banks in the same system is interbank payment;
d. Between two banks, the payment organization through correspondent bankingrelationship does
not need an additional bank as an intermediary;
26. Select the correct answers:
a. Minimum balance of debit card is 0;
b. Credit cardholders who pay late payments will incur high interest rates;
c. The supplementary card is issued when the main card is lost;
d. Credit cards cannot withdraw cash.
27.Choose the wrong sentence:
a. An anonymous check holder is the legal beneficiary;
b. Wire checks are non-transferable and non-cashable;
c. It is possible to transfer part of the amount stated on the check;
d. The drawer must still pay the amount stated on the check after the check is rejected by the
payee under the notice of suspension of payment;
28. Which of the following is not true about payment by letter of credit?
a. Buyers save time and effort in finding reputable and reliable partners;
b. The payment process is short, the parties proceed very quickly in making and checking documents;
c. Usually applied to import and export payment activities;
d. Protect the interests of both buyers and sellers of goods when they do not have the conditions to
directly exchange with each other.
29. Disadvantages of payment through correspondent banking relationship:
a. Does not facilitate money transfers, it will send payment orders settled through an organized agent
account with each other;
b. The payment involves 4 parties, the transaction step by step is not convenient;
c. Only within the banks of the same system, customers are not many;
d. Requires a modern and sensitive technology system when transacting between banks to ensure low
risk;
30.Using which of the following cards will increase your debit balance when buying goods and
services: a. Debit card;
lOMoARcPSD|36667950
b. Credit;
c. Payment card;
d. ATM card.
31.: Select the correct answers:
a. Payment cards are also known as debit cards;
b. The credit card only allows the account holder to withdraw funds within an agreed limit;
c. Checks are only used to withdraw cash at the bank;
d. Using a debit card means you are borrowing money from a bank.
32. Choose the wrong sentence, the customer wants to use the means of payment via the bank, the
customer must have:
a. Payment account;
b. Transaction deposit account;
c. Demand savings account;
d. Current account;
33. :The payment card has no embossed information?
a. The name of the card;
b. Card number;
c. Cardholder's ID number;
d. Cardholder's name;
34.A customer uses a card to buy goods at a supermarket, the value of goods is immediately debited to
the cardholder's payment account and credited to the supermarket's account. The card type and
payment method are:
a. Credit cards and electronic payment methods;
b. Debit cards and mechanical payment methods;
c. Credit cards and mechanical payment methods;
d. Debit cards and electronic payment methods;
35.Magnetic cards and electronic cards are distinguished from each other in:
a. Origin of the amount on the card;
b. Scope of card use;
c. Techniques for reading and storing information on cards;
d. Card payment method;
36.A customer uses a card inserted into the machine, after entering his personal identification number,
the customer withdraws an amount and his account is debited . What kind of card is it:
a. Card from;
b. Payment card;
c. ATM card;
d. Smart card;
37. Which of the following are characteristics of electronic payment methods:
a. Cardholders get deferred payment
b. The merchant's account will be credited after a certain number of days;
c. Used in conjunction with an electronic card;
d. There is no separation between transaction time and payment time; 38. When using a credit card,
the cardholder will have to:
a. Deposit an amount on the card account;
b. Bear interest if payment is not enough within a certain number of days;
c. Have a payment deposit account at a bank;
d. Payment immediately upon receipt of periodic statement from the bank;
39. Authorization for collection and authorization for expenditure are similar in that:
a. As a means of payment via bank issued by customers;
b. The document circulation process is similar;
c. There may be a risk of underpayment upon presentation of payment documents;
lOMoARcPSD|36667950
d. The drawer is the purchaser;
40. A customer uses a card to buy goods at a supermarket, the value of goods is immediately debited
to the cardholder's payment account and credited to the supermarket's account. The card type
and payment method are:
e. Credit cards and electronic payment methods;
Debit cards and mechanical payment methods;
Credit cards and mechanical payment methods; Debit cards and
electronic payment methods;
41. Check means:
a. Order slip issued by the purchaser;
b. Bill of exchange issued by the seller;
c. Debit note issued by the bank;
d. Special draft issued by the account holder;
42. Which of the following are characteristics of electronic payment methods:
e. Cardholders are entitled to deferred payment;
f. The merchant's account will be credited after a certain number of days;
g. Used in conjunction with an electronic card;
h. There is a network connection between payment participants;
43. Magnetic cards and electronic cards are distinguished from each other in:
a. Origin of the amount on the card;
b. Scope of card use;
c. Techniques for reading and storing information on cards;
d. Card payment method;
44. When using a credit card, the cardholder will have to:
a. Deposit an amount on the card account;
b. Bear interest if the minimum payment is not met after the specified time limit;
c. Have a payment deposit account at the bank;
d. Pay immediately upon receipt of periodic statement from the bank;
45. According to the Law on Transferable Instruments, if the time of force majeure or objective obstacles
is not taken into account, the check will only be paid if the check is presented within….
a. 15 days from the date of issuance;
b. 30 days from the date of issuance;
c. 45 days from the date of issuance;
d. 60 days from the date of issuance;
46. The typical risks in the collection order payment method are:
a. Fake payment vouchers;
b. The document arrives that the payer's account has insufficient funds to pay;
c. Debtor delays in making payment orders;
d. All sentences are correct;
47. The main uses of payment cards are:
a. Cash withdrawals;
b. Payment for goods and services;
c. Loan;
d. Payment to the bank;
48. Non-cash payment services in banks include:
b. Including payment service via payment account and not through customer's payment account;
c. Payment can be applied to State Treasuries in the same area;
d. Including payment services are collection orders, payment orders, checks and cards;
e. All right;
lOMoARcPSD|36667950
1. Choose the correct statement:
a. Account holders who are purchasers can use non-cash payment methods including security checks,
payment orders; letter of credit to pay for goods and services;
b. Merchant account holders can use non-cash payment methods including money orders, money orders
and letters of credit;
c. Individuals can use cash withdrawal cards (ATMs) to pay for goods and services anywhere;
d. All right;
2. According to current regulations, based on the amount used for payment, bank cards can be
classified into types:
a. Debit card, credit card, ATM card;
b. ATM card, credit card, overdraft card;
e. ATM card, credit card, check payment security card;
a. Prepaid cards, debit cards, credit cards;
61. What is the difference between a credit card and a debit card?
a. Payment uses of the card;
b. Scope of card use;
c. Card payment technology;
d. Origin of the amount on the card;
62. Which of the following statements does not describe the characteristics of the payment
authorization method:
a. The payment order goes from the debtor;
b. Suitable for payment for public goods used first, pay later;
c. The rotation process is consistent with the principle of debt first - credit later;
d. There are advantages for sellers;
63. Which of the following measures is the most effective to develop a highly secure and reliable
information infrastructure in the banking financial services industry:
a. Expand the supply of products and services;
b. Increase computing speed;
c. Expand sales with multiple supply channels;
d. Add new product applications;
64. : A business A in Ho Chi Minh City wants to pay for goods to a supplier in Hanoi, DN A should
use the following means:
a. Traveller's Cheques;
b. Bank card;
c. Authorization for collection;
d. Accreditative;
Sentences The similarities in check payment and payment order payment methods are:
a. Payment documents issued by the debtor;
b. The possibility of encountering fake vouchers;
c. Is transferred directly in the transaction;
d. All sentences are correct;
Which of the following statements is not true about non-cash payment intermediaries:
a. Not using cash but numbers on the books;
b. Not allowed to open payment accounts in foreign currencies;
c. Individuals who do not have full civil act capacity are not allowed to open accounts for transactions;
d. Payment card is one of the non-cash payment methods; Question: Choose the correct statement:
a. Debit and credit cards are both payment cards;
b. Debit card and credit card are both forms of lending according to the bank's credit limit;
c. Banks rely on debit and credit cards to assess the creditworthiness of their customers;
d. Debit and credit cards are not allowed to withdraw money at ATMs;
lOMoARcPSD|36667950
Question: Choose the correct statement from the following statements about the use of checks in
payment:
a. Banks and State Treasury may only sell blank checks to customers who have accounts at their units;
b. The beneficiary has the right to appeal in all cases where the check is rejected;
c. If the check is not paid, the assignor of the check is liable to the holder of the check;
d. Damaged checks must not be crossed out, but torn from the checkbook; Question: Choose the
incorrect statement from the following statements:
a. Payment services via customer's payment account, including: money transfer, collection and
payment;
b. Payment services not through the customer's payment account, including:
money transfer, collection and payment;
c. Non-cash payment service means a payment service that must go through a payment account;
d. Means of non-cash payment are electronic or paper vouchers;
Which sentence is not the same between payment by payment order and collection order?
a. There are 2 forms of payment through payment intermediaries;
b. Orders made by account holders must be made on pre-printed forms provided by payment service
providers;
c. Is a form of payment that the account holder makes an order, authorizes payment to payment service
providers to make payment;
d. Only used to pay different values, different periodicals;
Question : Arrange the following sentences in the correct order of payment:
1. Buyer and seller have a relationship of buying and selling goods and/or services with each other;
2. The buyer's bank pays the seller through the seller's bank;
3. The buyer writes a payment order to request his/her bank to pay for the purchase to the seller;
4. The seller's bank notifies the seller that the buyer has paid;a. 1, 2, 3, 4;
b. 1, 3, 4, 2;
c. 1, 3, 2, 4;
d. 1, 2, 4, 3;
Question : Which of the following statements is true?
a. Payment authorization made by the bank/service-providing organization;
b. The collection mandate is established by the beneficiary;
c. Payment authorization is often used to pay different values, different periods;
d. Collection mandates usually only pay for fixed amounts periodically;
Question: The form of payment is made between commercial banks in different systems and
locations, but each commercial bank has a deposit account at the other bank.
a. Inter-bank payment;
b. Payment through the State Bank;
c. Clearing;
d. Payment through correspondent banking relationship;
Question: Disadvantages of one-time payment via SBV
a. Slow payment speed;
b. Only applicable to banks in the same system;
c. Signature samples must be registered at the lead bank and other member banks; d. High cost of use;
lOMoARcPSD|36667950
Question: Which of the following statements is incorrect?
a. Anonymous check is a check that does not bear the beneficiary's name; and faceless checks are
transferrable only by endorsement;
b. Nominal check is a check that specifies the beneficiary's name; identification checks are not
transferable without endorsement;
c. Wire checks are non-transferable and non-cashable;
d. Cash checks paid in cash by the clearing bank;
Question 2: Providing payment service via account is:
a. Provision of means of payment and credit granting;
b. Provision of payment means and financial leasing;
c. Provision of payment means and securities brokerage;
d. Provide payment means and execute payment orders;
Question : The disadvantages of payment by Collection Order are:
a. Cannot be used in the case of payment for services provided with an accurate means of measurement;
b. The collection order is made by the beneficiary, so there may be a delay in the payment process ;
c. Using the Collection Mandate takes a lot of time and costs;
d. Collection mandate is only used in transactions of goods/services of a public nature;
Question : The advantages of the form of clearing between banks are:
a. The payment range is wide, nationwide because the network of branches of the
State Banks covers all localities across the country;
b. If banks have to pay but the balance is not enough to pay, the State Bank and commercial banks will
lend to supplement the shortfall in clearing;
c. Can quickly process many payment transactions at the same time;
d. It is possible to process many transactions between banks that have payment relations at the same
time;
Question : Which of the following is not a feature of a debit card?
a. Withdraw money directly from your account to make the payment;
b. Ability to withdraw money from ATMs;
c. Credit balance of card account decreases upon payment;
d. The debit balance of the account increases upon payment;
Question: Common characteristics of all payment methods between banks are:
a. No cash;
b. Execute transactions one by one;
c. Performing multiple transactions at the same time;
d. Nationwide payment coverage;
Sentence Differences between the Czech Republic and the payment authorization:
a. Payment order maker;
b. Payment order recipient;
c. Time of handing over documents;
d. Beneficiaries of vouchers;
Question: What is the risk in the payment method by collection order:
a. Fake payment vouchers;
b. Insufficient funds in the payer's account when the documents arrive;
c. Debtor delays in making payment orders;
d. Creditors are late when making payment orders;
Which of the following statements is incorrect when talking about the utility of payment services
via NH:
a. Reduce costs of transportation, storage, cash inventory;
lOMoARcPSD|36667950
b. Avoid all possible risks;
c. Effectively combat tax loss;
d. To be provided with banking services with many incentives;
Question Which of the following methods of collecting electricity service charges brings the most
benefits and advantages to a water company?
a. Toll collection at home;
b. Collect fees through collection mandate;
c. Collect fees through payment authorization;
d. Receive payments by check;
Question Which form of payment between banks has the most advantages when making
payments between banks in the same system: a. Inter-bank payment;
b. Inter-bank payment;
c. Payment via agent bank;
d. Clearing;
Question Characteristics of clearing between banks are:
a. Can process multiple payment transactions at the same time;
b. Slow payment speed;
c. Only applicable to banks in different systems;
d. The member banks are not required to open an account at the leading bank; Question What is the
BIN number on the ATM card?
a. Cardholder's personal identification number;
b. Code indicating the cardholder's transaction account;
c. Card issuing bank's code;
d. The card's password is provided by the bank upon issuance;
Which of the following statements is not true about SWIFT:
a. The fast speed allows to process a large number of transactions in any country in the world;
b. Very high security and safety;
c. The cost for an electric transaction is lower than the traditional means of communication;
d. Using SWIFT will follow a uniform standard;
Question When there are multiple checks filed in the bank at the same time to collect money from
a drawer but the drawer's ability to pay is not enough to pay all of those checks, the order of
payment will be:
a. In order of check number and draw date;
b. By date of draw and order of check number;
c. According to the value of the amount on each check;
d. NH will not pay all such checks;
Question Currently, among the non-cash payment means, which means of payment has the largest
total transaction volume in Vietnam?
a. Checks;
b. Accreditative;
c. Authorization for collection;
d. Bank card;
Which of the following statements about checks is true :
a. Past the validity period of the check, in order to receive payment, the beneficiary must request the
drawer's bank to pay;
b. When a check has been ordered, the payee is responsible for paying the amount stated on the check
even if the time limit for presentation of the check has expired;
c. The buyer of the traveler's check that signs the check can convert it into cash equivalent to the amount
written on the check by the buyer;
d. For crossed checks, which are non-cashable checks; The advantage of collection mandate is:
a. Can be used for both domestic and international payments;
b. The bank can control the solvency of the payer;
lOMoARcPSD|36667950
c. The payment process is fast but complicated;
d. Reduce costs for banks;
Question Suppose you are a payment officer, when receiving a payment order from a customer,
what should you pay attention to first?
a. The beneficiary does not have an account at the bank;
b. The payer does not have an account at the bank;
c. Beneficiary does not have enough balance in the bank account;
d. The payer does not have enough balance on the bank account;
Sentence
Which of the following is true about spending and revenue authorizations?
A. Payment authorization and collection authorization are both established by the bank
(Wrong, the payment order and the collection order will be made by the person who needs to pay or
collect money, who is not always the bank.)
B. When using a payment order and a collection order, the buyer and seller must have an account at the
same bank
(Wrong, for a payment order, the buyer and seller do not necessarily have the same bank account.
For a collection order, the buyer and seller must both have a bank account (same or different bank) )
C. Remittance service fee to be paid by the payer or the beneficiary ( Yes, depending on the agreement,
the payer or the beneficiary will pay the bank transfer fee)
D. Beneficiary's bank receives remittance fee
(Wrong, the money transfer fee will be borne by the payer's bank) Sentence is used between:
a. Banks in the same system, same location, regular transactions;
b. Banks in the same system, different locations, infrequent transactions;
c. Banks are different from the system, in different locations, with regular transactions;
Banks are different from the system, in the same area, with irregular transactions;
d.
Sentence In payment of checks, the person whose checking account is debited:
a. Beneficiary;
b. Drawer;
c. Transferor;
d. Payee;
Question Type of check guaranteed to be paid by the bank to the beneficiary by freezing
the check issuer's amount at the bank is:
a. Security check ;
b. Cash checks ;
c. Check slash ;
d. Check by order ;
Crossed check sentence :
a. Beneficiaries do not need to have an account at NH ;
b. The issuer does not need to have an account at NH ;
c. The issuer must have an account at the bank ;
d. The beneficiary must have an account at the bank ;
The sentence “…………………….. is an unconditional payment order, signed by a bank customer,
ordering the bank to deduct a certain amount of money from his account to pay. to the
designated person”.
a. Promissory Note (Promissory Note) ;
b. Bill of Exchange (Bill of Exchange) ;
c. Check (Check) ;
d. Letter of Credit (L/C) ;
The sentence "Act of commitment to pay by the person obligated to pay when the bill of exchange
reaches maturity" is a transaction:
e. Acceptance of bill of exchange
lOMoARcPSD|36667950
f. Endorsement of bills of exchange
g. Guarantee of bills of exchange (Aval)
h. Protest (Protest)
Question Account holders with payment deposits at banks can use:
a. Checks, credit cards, payment orders, collection orders;
b. Checks, ATM cards, payment orders, collection orders;
c. Checks, credit cards, ATM cards, payment orders;
d. Checks, promissory notes, credit cards, ATM cards;
Question If the time of force majeure event is not counted, the check will only be paid if the check
is presented within:
a. 10 days from the date of issuance;
b. 20 days from the date of issuance;
c. 60 days from the date of issuance;
d. 30 days from the date of issuance;
When paying by payment order submitted by the customer to the bank, the payment staff should
pay attention to the following problems: a. The beneficiary does not have a bank account;
b. The payer does not have an account at the bank;
c. The beneficiary does not have enough balance on the account at the bank;
d. The payer does not have enough balance on the account at the bank;Sentences The
mode of payment for letters of credit is almost exclusively used in payment of
import-export contracts and is rarely used to pay for domestic goods sale and
purchase contracts because: a. Domestic customers are limited in foreign languages;
b. The payment procedure is too complicated;
c. Customers have no need to use this method in payment of domestic sales contracts;
d. The importer is not fluent in the language of the exporter;Sentence Payment cards can be used
for: a. Any debt payments to the bank;
b. Any payment for services accepted by the beneficiary;
c. Any payment accepted by the beneficiary;
d. Any payment for goods accepted by the beneficiary;
Sentence Element not required when issuing bill of exchange a. Place of drawing of bills
of exchange;
b. An unconditional order to demand money;
c. Amount in figures and words of the bill of exchange;
d. Title of bill of exchange;
Which of the following statements is most accurate:
a. Payment via bank is a type of money receiving service, whereby the bank performs the receipt of
money from the account of the payer, by debiting the account, to the account of the beneficiary, by
credit the account;
b. Payment via bank is a type of money receiving service, whereby the bank makes a deduction to
transfer money from the account of the payer, by debiting the account, to the account of the
beneficiary, by how to credit the account;
c. Payment via bank is a type of payment service whereby the bank makes a deduction to transfer money
from the account of the payer, by crediting the account, to the account of the beneficiary, by how to
debit the account;
d. Payment via bank is a type of payment service whereby the bank makes a
deduction to transfer money from the account of the payer, by debiting the account, to the account of
the beneficiary, by how to credit the account; The mandator's sentence is:
a. Buyers and sellers of goods;
b. Buyers of goods and services;
c. Person with balance in a checking account;
d. Sellers of goods and services;
lOMoARcPSD|36667950
The difference between a debit card and a credit card is:
a. Credit cards are used to grant credit, debit cards are used to pay debts
b. Credit cards do not require customers to have money on their accounts to use,debit cards require
customers to have money on their accounts to use;
c. Credit cards issued by credit institutions, debit cards issued by banks:
d. Credit cards require customers to have money on their accounts to be used, debit cards do not
require customers to have money on their accounts to be used;
Sentence The difference in usage between blank endorsement and command endorsement is:
a. Endorsement blank for the purpose of transferring the bill of exchange by hand, endorsing
to order for the purpose of transferring the draft draft to the order of the beneficiary;
b. Endorsement is blank for the purpose of facilitating the transfer of bills of exchange,
endorsing by order is for controlling the transfer of drafts;
c. Endorsement is blank for the purpose of facilitating the transfer of drafts, endorsing by
order is for the purpose of terminating the transfer of drafts; d. The endorsement is blank for the
purpose of facilitating the transfer of the bill of exchange, the endorsement by order is for the
purpose of restricting the transfer of the bill of exchange;
The characteristics of the documentary credit payment method are stated as follows:
a. Documentary credit payment method, the importer commits to pay the exporter's bank;
b. Documentary credit payment method, the importer's bank commits to pay theexporter's
bank;
c. Documentary credit payment method, the importer's bank commits to pay the exporter;
d. Documentary credit payment method, the importer commits to pay the exporter;
Sentences of the method of payment by collection with documents under D/P condition are
stated as follows:
a. Payment method by collection with documents under D/P terms, the exporter cannot deliver
the goods without receiving the money from the importer;
b. Payment method by collection with documents under D/P terms, the exporter can receive
money without delivering the goods to the importer;
c. Payment method by collection with documents under D/P terms, importer cannot receive
goods without paying exporter;
d. Payment method by collection with documents under D/P terms, the exporter can receive the
goods without paying the exporter;
Sentences of the method of payment by collection with documents under D/A condition are
stated as follows:
a. The exporter cannot deliver the goods without receiving payment from the importer;
b. The exporter can receive money without delivering the goods to the importer;
c. The importer cannot receive the goods without paying the exporter;
d. The importer can receive the goods without paying the exporter;
Sentence According to the current regulations of Vietnam, the time limit for presenting a request
for payment of a check is ....., from the date of issuance:
a. 30 days;
b. 60 days;
c. 45 days;
d. 6 months;
Question Payment of checks shall be terminated after ……., from the draw date indicated on
the check:
a. 30 days;
b. 60 days;
c. 45 days;
d. 6 months;
If the check is presented after the time limit for presentation for
lOMoARcPSD|36667950
payment but not more than ………… from the date of drawing, the drawee can still pay if the
drawee does not receive the notice of suspension of payment for check and the drawer has
sufficient funds on the account to pay.
a. 30 days;
b. 60 days;
c. 45 days;
d. 6 months;
Which of the following cards can be used by customers when their deposit accounts are
exhausted:
a. Debit card;
b. Escrow card;
c. Card used by overdraft method;
d. Credit;
Sentence In payment of a check, the drawee is:
a. The person who is obligated to pay;
b. The person serving the payer;
c. Beneficiary of the check;
d. The person serving the payee of the check;
Which of the following means of payment is no longer used in Vietnam:
a. Bill of exchange for debt receipt;
b. Bill of exchange for debt collection;
c. Payment check;
d. Accreditative;
Which of the following states the difference between a check and a payment order:
a. Subjects make different payment orders;
b. The probability of counterfeit vouchers in checks is higher than in payment orders;
c. Checks are transferred directly in the transaction, while the payment order is not;
d. Sentences b and c are correct; Bank Card is used to:
a. Cash withdrawals
b. Payment for goods and services
c. Bank loan
e. Both a and b are correct
Sentence Distinguish between magnetic cards and electronic cards in: b. Origin of the card;
c. The type of card reader suitable for it;
d. Scope of card use;
e. Methods of immediate or deferred payment;
Question Based on the technology of the card, the bank's cards include:
f. Magnetic and chip cards;
g. Magnetic and credit cards;
h. Magnetic cards and ATM cards;
i. Chip cards and ATM cards;
Question Payment by check will
f. Faster than paying by proxy;
g. Slower than paying by ATM;
lOMoARcPSD|36667950
h. All forms of payment are the same;
i. Not enough grounds to conclude the above ideas;
Sentence The method of payment between customers via bank is
a. The Bank deducts from the payer's account to the beneficiary's account through the Bank's operations;
b. The Bank deducts from the bank's account and transfers it to the payer's account to pay the
beneficiary;
c. The bank deducts from the beneficiary's account and transfers it to another bank related to the
customer in payment;
d. All of the above are true;
Question According to current regulations, the amount used for debit card payment is from the
account:
a. Term deposit account;
b. Payment deposit account;
c. Demand savings account;
d. Current account;
Which of the following check types is not subject to a time limit for payment: a. Cash check
b. Traveller's Cheques
c. Security check
d. Check slash
Which of the following statements does not describe the characteristics of the payment
authorization method:
a. The payment order goes from the debtor;
b. Suitable for payment for public goods used first, pay later;
c. The rotation process is consistent with the principle of debt first - credit later;
d. There are advantages for sellers;
Which of the following characteristics best describes the nature of the collection mandate?
a. Used for goods transactions between two parties that have regular relations;
b. The payment order goes from the debtor;
c. The process of circulating documents is like a check;
d. Buyers have the advantage of being paid late;
Question When commercial banks provide good payment facilities, they will attract:
a. Source of savings deposit
b. Source of non-transactional deposits
c. Source of transaction deposit
d. All sources above
Question To use the means of payment via bank, customers must have:
a. Current account at the bank
b. Term savings account at the bank
c. Payment deposit account at the bank
d. a and c are correct.
Part 3
Which business sentence is an on-balance sheet transaction of NH?
a. Buy corporate bonds;
b. Advise;
lOMoARcPSD|36667950
c. Investment Management;
d. Insurance;
Question 1. SMALL financial services
A. Increase the capital source of the bank;
B. Decreasing the bank's assets;
C. It both increases capital and reduces the bank's assets;
D. Does not change the capital and assets of the bank.
Question 2. What rights does the trustee have over the trust property?
A. Possession;
B. Possession, use; C. Settlement;
D. Possessing, using, a part of the right to dispose.
Question 2. In January/N, enterprise A has 15,000 USD. Currently, enterprise A has a capital need
of 300 million VND for production and a debt of 10,000 USD to be paid at the beginning of
February/N. What should businesses do to both have money for production and can eliminate
exchange rate risk when paying debts in February/N (knowing that the USD exchange rate is
trending up)?
A. Bank loan of 15 000 USD with floating interest rate for 1 month term;
B. Bank loan of 15 000 USD with interest rate of 1.7%/year for 1 month term;
C. Sign a foreign currency swap contract worth USD 15 000;
D. Deposit 15,000 USD in the bank with the interest rate of 0.25%/year for 1 month term.
Question 1. Choose FALSE when talking about derivative services:
A. Derivatives are a type of financial hedging;
B. Derivative services originate from potential risks in economic contract transactions;
C. Derivative services are always born before a contract transaction;
D. Derivatives services help clients to hedge and minimize financial risks in addition to profit-making
speculation.
Question 2. What are the characteristics of financial services?
A. Revenue from fees, not using capital, on-balance sheet activities;
B. Revenues from fees and interests, capital use, off-balance sheet activities;C. Income from
interest, not using capital, on-balance sheet activities;
D. Income from fees, not using capital, off-balance sheet activities.
Question 1. Services of modern banks are:
A. Accepting deposits;
B. Pay;
C. Commission;
D. Preservation, consignment.
Question 2. Choose the wrong sentence:
A. Entrustment contracts are not revocable;
B. The trust maker may retain only part of the control over the assets;
C. Liquidation of assets under a will is a form of trust;
D. In Vietnam, individuals, banks and financial institutions are entitled to entrust capital to invest.
Question 1. Choose FALSE. Receive entrustment from individual customers including:
A. Liquidation of assets according to the will;
B. Securities trading;
C. Operate individual trusts under contract;
D. Property guardian.
Question 1. What is NOT a consulting service?
A. Low level of risk;
B. Requires a team of banking consultants to be reputable, knowledgeable and professional;
C. Content can be investment advice, purchase and sale, consolidation, merger;
lOMoARcPSD|36667950
D. The consultant is responsible for the decisions that the consultant makes. Question 2. Choose FALSE:
A. In the trust service, almost all rights on the property are transferred by the owner to the trustee,
retaining only a part of the control of the property;
B. In the representation service, the owner still retains the right over the property, only transferring to
the representative the establishment and performance of a
number of civil transactions within the scope of competence;
C. In practice, trust contracts are often revocable;
D. There are three main types of agency services: contract representation, asset management, and
litigation.
Question 1: Which of the following is a type of bank guarantee issued by a credit institution to the
guarantor to ensure the obligation to return the advance payment received by the customer to the
guarantor under the signed contract: A. Payment Refund Guarantee;
B. Payment guarantee;
C. Bid guarantee;
D. Loan guarantee.
Verse 2: Choose the FALSE sentence. The guarantee contract or guarantee commitment of the
guarantor must be signed by:
A. Legal representative;
B. Underwriting risk manager; C. Guarantee appraiser;
D. Guaranteed person.
Question 1. Choose the TRUE statement about Trust Services:
A. In Vietnam, only commercial banks are allowed to perform entrustment services;
B. In trust services, the trustee (the trustee) does not have the right to possess, use and partially dispose
of the property;
C. Credit institutions or foreign bank branches are authorized by individuals;
D. The entrustor must bear the risk when performing entrustment operations in the field related to
banking activities.
Question 2. Banks are not authorized to entrust or receive entrustment in the field of business:
A. Asset Management
B. Insurance business;
C. Banking activities;
D. Buying and selling real estate.
Verse 1 . Which of the following statements false:
A. Financial services include any banking and other financial services (except insurance);
B. When the bank performs financial services, it will not change either side of thebank's balance sheet;
C. Financial services help banks supplement profits and spread risks in business;
D. Financial services in banking are services associated with banking activities.Question 2. In what way
is the guarantee by the method of selling all or nothing different from the other guarantee methods?
A. Commitment to buy all issued securities regardless of whether they are fully distributed or not;
B. If the amount of securities sold reaches a ratio lower than the required rate, the entire issuance will
be canceled;
C. If the issued securities are not sold out, the entire issuance will be canceled and the money sold will
be owned by the underwriter;
D. If the issued amount of securities is not sold out, the entire issuance will be canceled and the money
sold will be returned to the buyer.
Question 1. Which financial services do commercial banks not need to establish or acquire
subsidiaries or affiliates?
A. Underwriting securities;
B. Consulting services;
C. Insurance;
lOMoARcPSD|36667950
D. Financial leasing.
Question 2: Choose the TRUE statement:
A. In preservation and deposit services, the Bank still has to pay interest to customers if the preservation
is money;
B. Futures contracts are still traded on the OTC market;
C. An options contract does not oblige its buyer to execute the committed transaction, but an option
premium is required in all cases.
D. Large-scale financial leasing companies are still allowed to contribute capital, buy shares, establish
subsidiaries and associate companies.
Question: The similarity between agency service and fiduciary service is:
a. The owner retains the rights on the property;
b. Complex work, depending on the customer;
c. High legality, strictly regulated by the provisions of law;
d. Can be terminated by simple procedures;
Question : Which is not a primary agency job:
a. Contract signing representative;
b. Management representative;
c. Legal representative;
d. Personal tax processing representative;
Question: Which of the following is not true about the service of preserving valuables:
a. Items sent can be gold, money, souvenir photos;
b. The Bank must return the consignment intact;
c. Preserved money and gold are charged interest;
d. The bank knows the details of the items deposited;
Question: Financial services bring revenue to banks from:
a. Credit interest;
b. Fee collection;
c. Financial investment;
d. Mobilizing deposits and providing payment services;
Question 1: In a trust service, what rights do the trustee have over the trust assets:
a. Right of possession and right of use;
b. Right of control and right of disposition;
c. Possession only:
d. Nearly all rights as an owner to property;
Question 2: What rights does the trustee have with respect to the trust property: a. Partial control;
b. Part of the right to use;
c. Partial decision-making power;
d. Have all the rights of 1 owner to the property;
Question 3: According to Circular 04/2012 TT_NHNN, credit institutions, foreign bank branches are
not allowed to receive entrusted capital from any of the following entities:
a. Other credit institutions;
b. Overseas organizations;
c. Individuals;
d. Domestic enterprises;
Question 2: Choose the wrong sentence: The bank can perform the operations related to the treasury
as follows:
a. Collecting and paying on behalf of;
b. Transfer money at home and abroad;
c. Currency exchange, money counting;
d. Receive entrustment from customers;
lOMoARcPSD|36667950
Question: Choose the wrong sentence:
a. Commercial banks are entrusted with legal entities;
b. Custody and deposit services are classic banking services;
c. Leasing safes, safes belonging to financial leasing services;
d. Bank guarantee is a form of credit extension;
Question: Which service belongs to the banking entrustment service:
a. Cash management;
b. Identify an efficient capital structure;
c. Entrust to issue securities for enterprises;
d. Consulting in financial management;
Which of the following services is not part of treasury services of commercial banks :
a. Exchange money at the request of the customer ; b Liquidation
of assets according to the will ;
c.Cash collection at the customer's location ;
d. Remittance abroad ;
Which of the following transactions is not part of treasury services?
a. Transfer money at home and abroad ;
b. Open a payment deposit account ;
c. Exchange money ;
d. Household collection ;
Which of the following banking operations only collect fees, not interest?
a. Factoring ;
b. Loans ;
c. Buy government bonds ;
d. Payment via bank ;
Question What is the method of underwriting of securities that the underwriters follow the
instructions of the issuer to sell all of the securities, otherwise, the entire issuance will be
canceled? a. Guarantee with the highest commitment;
b. Guarantee with firm commitment;
c. Guarantee under the minimum - maximum method;
d. Guarantee in the form of either the sale of all or nothing;
Question A customer deposits gold and has to pay fees to the custodian bank. The relationship
between the bank and the customer is:
a. The relationship between the borrower and the lender;
b. The relationship between the consignee and the depositor;
The relationship between the lessor and the lessee of the safe; Owners and agents;
The service of cash payment from Customer's account to the address requested by Customer is:
a. Payment service using UNT;
b. Recurring UNC billing service;
c. Treasury services;
d. Account management services and safes; Trustee functions:
a. Master asset management;
b. Manage assets for the benefit of beneficiaries;
c. Preserving valuables;
d. a and b;
Which of the following is not true about commercial deed?
a. Commercial deed is called commercial paper;
b. Usually issued by reputable large enterprises;
c. Enterprises that want to issue commercial certificates need collateral;
d. For businesses with a low credit rating that want to issue commercial certificates,
lOMoARcPSD|36667950
a bank guarantee is required;
Which statement is not true about agency services:
a. The relationship between the customer and the representative is not as close as in entrustment;
b. An authorized representative on behalf of the customer in each specific case;
c. The agent has more rights over the TS than the fiduciary service; d. All is incorrect;
Which of the following measures is the most effective to develop a highly secure and reliable
information infrastructure in the banking financial services industry:
e. Expand the supply of products and services;
f. Increase computing speed;
g. Expand sales with multiple supply channels;
h. Add new product applications;
Question What form does the money transfer service take?
a. Investment profession;
b. Foreign currency trading operations;
c. Entrustment business;
d. Consulting profession;
In trust services, which party is the profit and loss business results calculated for:
a. Bank;
b. Client;
c. If there is a loss, the bank will bear, the profit will be enjoyed by the customer;
d. If there is a loss, the customer will bear it, and the bank will enjoy the profit; Question Banks provide
consulting, entrustment, financial investment, gold and silver trading services mainly for the
following purposes:
a. Reduce operational risks;
b. Capital mobilization;
c. Increased liquidity;
d. Diversify revenue sources;
A bill of exchange accepted by a bank is considered a highly secure instrument because:
e. The bank is the person responsible for securing the payment to the beneficiary;
f. Issued and guaranteed by the Government;
g. Low interest rate should be high safety;
h. Be committed by the importer to repay the debt;
Question: Collecting, paying for, transferring money, changing money... What is the service? a.
Commission;
b. Funds;
c. Representative;
d. Advise;
Which of the following is not true about Banker's Acceptance?
a. Not transferable ;
b. For a bank accepting a bill of exchange, agreeing to seal the acceptance is considered a form of
signature credit for the customer who is the issuer ;
c. A bill of exchange issued by an enterprise ;
d. Be stamped "accepted" by the bank on the draft ;
Commercial banks are NOT authorized to entrust, receive entrustment, or agent in the field
related to:
a. Banking activities;
b. Insurance Business;
c. Manage assets in accordance with regulations of the State Bank;
d. Personal trust;
Question Is the consultant legally responsible for the decisions made by the consulted client?
a. Take full responsibility;
lOMoARcPSD|36667950
b. Take part of the responsibility;
c. Responsibilities will be divided in two;
d. No liability;
Question The team of consulting service department of commercial banks is usually:
a. Any employee working in the NH;
b. Professional, experienced and knowledgeable staff in the fields of consulting services;
c. Senior staff with good achievements in the bank's activities;
d. Elderly employees have worked for a long time and are trusted by everyone;
Question What is the positive impact of diversification of banking services on commercial banks?
a. Risk dispersion; promote the joint development of professions;
b. Increase profits while increasing service expansion costs;
c. Serving customers in a package method, improving quality for customers;
d. Open more branches, easily redirect business; Sentence Functions of trust:
a. Embrace asset management
b. Manage assets for the benefit of beneficiaries
c. Preserving valuables
d. a and b
Which of the following is not true about the bank safe deposit box service?
a. The procedure is very quick;
b. Assets are guaranteed to be absolutely safe;
c. The cost of using this service is lower than renting a safe;
d. Assets stored in this service are profitable;
Which of the following is not a financial service:
a. Counting money;
b. Guardianship of property;
c. Accreditative;
d. Customer consulting;
Which of the following statements is incorrect about the utility of preserving valuables and
valuable papers at a bank:
a. Customers are not allowed to check the property during the preservation period to ensure safety and
secrecy;
b. The Bank commits to keep confidential information about the customer's TS;
c. To terminate the rental contract of the locker ahead of time;
d. To authorize another person when there is a written authorization in accordance with the law;
Which of the following statements is false about the preservation of assets at a bank?
a. The bank must preserve the absolute safety of the customer's assets. The property must be kept intact
until it is returned to the customer;
b. The bank must be responsible for the legal ownership of the assets deposited
with the bank for preservation;
c. Customer is responsible for fully and timely payment of preservation fee to the bank in accordance
with regulations;
d. If the property box is returned with its lock and seal intact, the customer must be
responsible for the entire quantity and quality of the property contained in the iron box;
Which of the following is a representative service of a bank: a. Payment of wages to
the customer's employees;
b. Distribution of assets on the will;
c. Consulting on real estate purchase and sale;
lOMoARcPSD|36667950
d. Managing the offices of BC company;
Which of the following sentences is correct:
a. A trust is a trustee who has the right to hold and manage such assets in perpetuity since the death
of the trustee;
b. The representative has the right to perform certain tasks if the client sends a letter of appointment;
c. For fiduciary services, the trustee has no right to dispose of the assets;
d. The representative has a part of the right to dispose of the client's property;
Question In addition to areas related to banking activities, commercial banks are entitled to
entrust, receive entrustment, and act as agents in related fields?
a. Rental Property;
b. Insurance Business;
c. Buying and selling real estate;
d. Securities investment;
Sentence Choose the correct sentence, the "open" preservation method is:
a. Open a savings account at the Bank.
b. The method has high profitability for customers.
c. The form of money or gold is not profitable during the deposit period.
d. Preserve valuables.
Question Choose the correct answer:
a. Leasing safes to customers is an “open method of storage in storage and deposit services.
b. The maintenance fee for one box of property for one year is 0.5% of the property's value.
c. The service of preservation and consignment was born to ensure the safety and secrecy of assets
for customers.
d. When the customer receives back the asset box with the lock and seal intact outside the box but the
entire quantity and quality of the assets contained in the box is affected or changed, the bank must
be responsible. The essence of guarantee services in banks:
a. A special type of credit;
b. A service for the purpose of promoting the bank's brand;
c. The Bank is the third party confirming the contract between the two established transaction parties;
d. Reducing risks in banking business;
Why is it said that custodial and depository services are a classic type of banking service:
a. Because this is the service that brings the bank the most revenue in the services.
b. Because this is a service that appeared right from the time the bank was born in history.
c. Because customers often go to the bank to ask the bank to perform this service the most.
d. Because this is a service available in all banks.
Which of the following is not true for the benefit of customers when using the service of
preserving valuables:
a. The bank's valuable assets are absolutely safe;
b. Saving space and costs of purchasing equipment to store assets and papers;
c. Periodically check the value of assets by the bank;
d. To benefit from assets during the time of the bank's preservation; Question: Why do customers use
internet banking system in your opinion?
a. Because customers save time in querying account information and records of transactions of the
day;
b. As customers can make all unlimited payments quickly and conveniently;
c. Because customers can make loans right on the system;
d. Because customers can pay service bills within the allowable limit;
D: For the payment of service bills through internet banking, banks do not limit the amount for
customers.
lOMoARcPSD|36667950
Which of the following are the direct benefits of guarantee services for banks:
a. Bring a large number of transactions;
b. Bring in a source of income;
c. Enhance the reputation and brand of the bank;
d. Easy implementation of customer policy;
Question Difference between investment trust and term savings:
a. When the contract expires, customers who want to continue to entrust their investment do not need
to go through the procedures again;
b. The interest rate on investment trust is higher than that of term savings;
c. Savings deposits have a higher risk than investment trusts;
d. There is no difference, just different names;
Question 1. SMALL financial services
E. Increase the capital of the bank;
F. Reducing the assets of the bank;
G. It both increases capital and reduces the bank's assets;
H. Does not change the capital and assets of the bank.
Question 2. What rights does the trustee have over the trust property?
E. Possession;
F. Possession, use; G. Settlement;
H. Possessing, using, a part of the right to dispose.
Question 1. Choose the TRUE sentence when talking about arbitrage trading and arbitrage
speculation:
A. These two forms are essentially one because they are used to enjoy exchange rate
differences;
B. These two forms are subject to exchange rate risk and require a certain amount of capital;
C. The administrator can determine the profit level due to the exchange rate difference from
these two forms;
D. Currency arbitrage takes place at the same time while arbitrage speculation takes place at
two different times.
Question 2. In January/N, enterprise A has 15,000 USD. Currently, enterprise A has a capital need
of 300 million VND for production and a debt of 10,000 USD to be paid at the beginning of
February/N. What should businesses do to both have money for production and can eliminate
exchange rate risk when paying debts in February/N (knowing that the USD exchange rate is
trending up)?
A. Bank loan of 15 000 USD with floating interest rate for 1 month term;
B. Bank loan of 15 000 USD with interest rate of 1.7%/year for 1 month term;
C. Sign a foreign currency swap contract worth USD 15 000;
D. Deposit 15,000 USD in the bank with the interest rate of 0.25%/year for 1 month term.
Question 1. Choose FALSE when talking about derivative services:
A. Derivatives are a type of financial hedging;
B. Derivative services originate from potential risks in economic contract transactions;
C. Derivative services are always born before a contract transaction;
D. Derivatives services help clients to hedge and minimize financial risks in addition to profit-making
speculation.
Question 2. What are the characteristics of financial services?
A. Revenue from fees, not using capital, on-balance sheet activities;
B. Revenues from fees and interests, capital use, off-balance sheet activities;C. Income from
interest, not using capital, on-balance sheet activities;
lOMoARcPSD|36667950
D. Income from fees, not using capital, off-balance sheet activities.
Answer: EASY
Explanation : Financial services provide banks with revenue from fees, not from interest, like credit
or investment. In most financial services activities, banks often do not use capital, but only take
advantage of available facilities and advantages from experience. Because it is an off-balance sheet
activity, when the bank performs these activities, it does not change the two sides of the bank's
balance sheet.
LE TRIEU VU
MSSV: 030326100308
Question 1. Services of modern banks are:
E. Accepting deposits;
F. Pay;
G. Commission;
H. Preservation, consignment.
Answer:
Explanation: The services of deposit receipt, payment and preservation and deposit are traditional
banking services, which have existed for a long time. Trust service is one of the services of a modern
bank, playing an increasingly important role and generating more profits for the bank. Question 2.
Choose the wrong sentence:
E. Entrustment contracts are not revocable;
F. The trust maker may retain only part of the control over the assets;
G. Liquidation of assets under a will is a form of trust;
H. In Vietnam, individuals, banks and financial institutions are entitled to entrust capital to invest.
Answer: EASY question
Explain:
- According to Circular 04/2012/TT-NHNN on the profession of trusteeship and trusteeship of
organizations
NGUYEN NGOC THAI NGUYEN
MSSV: 030326100151
Question 1. Choose FALSE. Receive entrustment from individual customers including:
E. Liquidation of assets according to the will;
F. Securities trading;
G. Operate individual trusts under contract;
H. Property guardian.
Answer: REMOVE
Question 2. Choose FALSE . Financial leasing activities must meet one of the following conditions:
A. At the end of the lease term under the contract, the lessee is entitled to transfer the ownership of the
leased asset or continue to lease it as agreed by the two parties;
B. At the end of the lease term under the contract, the lessee has the right to preemptively purchase the
leased asset at a nominal price lower than the actual value of the leased asset at the time of
acquisition;
C. The lease term of an asset must be at least 60% of the time required to depreciate the leased asset;
D. The total rental amount for an asset specified in the finance lease contract must be at least equal to
the value of that asset at the time of receiving the leased asset.
Answer: EASY
Explanation : The total rent for an asset specified in a finance lease contract must be at least equal to
the value of that asset at the time of signing the contract. (Article 133 of the Law on Credit
Institutions).
DIEP THANH DANH
MSSV: 030326100039
lOMoARcPSD|36667950
Question 1. What is NOT a consulting service?
E. Low level of risk;
F. Requires a team of banking consultants to be reputable, knowledgeable and professional;
G. Content can be investment advice, purchase and sale, consolidation, merger;
H. The consultant is responsible for the decisions that the consultant makes.
Answer: EASY
Explanation : Legally, the consultant is not responsible for the decisions made by the consultant.
Because this service only provides information and best advice on the areas that customers are
interested in, they need to learn to help customers make accurate, effective decisions and minimize
risks. And the final decision still belongs to the customer and they are the ones who make that
decision, so they are responsible for the results.
Question 2. Choose FALSE:
E. In the trust service, almost all rights on the property are transferred by the owner to the trustee,
retaining only a part of the control of the property;
F. In the representation service, the owner still retains the right over the property, only transferring to
the representative the establishment and performance of a
number of civil transactions within the scope of competence;
G. In practice, trust contracts are often revocable;
H. There are three main types of agency services: contract representation, asset management, and
litigation.
Answer:
Explanation : Trust has a very high legal status, which is strictly governed by
legal regulations. The trustee holds the right to property, possession of the property has a legal
basis, the trustee must comply with the terms of the trust contract or court decisions.
VUONG HOANG ANH
: 030326100018
Question 1: Which of the following is a type of bank guarantee issued by a credit institution to the
guarantor to ensure the obligation to return the advance payment received by the customer to the
guarantor under the signed contract:
E. Payment Refund Guarantee;
F. Payment guarantee;
G. Bid guarantee;
H. Loan guarantee.
Verse 2: Choose the FALSE sentence. The guarantee contract or guarantee
commitment of the guarantor must be signed by:
E. Legal representative;
F. Underwriting risk manager;
G. Guarantee appraiser;
H. Guaranteed person.
Answer: EASY
According to Clause 1, Article 15, “Circular on regulations on bank guarantees” No. 28/2012/TT-NHNN
lOMoARcPSD|36667950
Question 1. Choose the TRUE statement about Trust Services:
E. In Vietnam, only commercial banks are allowed to perform entrustment services;
F. In trust services, the trustee (the trustee) does not have the right to possess, use and partially dispose
of the property;
G. Credit institutions or foreign bank branches are authorized by individuals;
H. The entrustor must bear the risk when performing entrustment operations in the field related to
banking activities.
Question 2. Banks are not authorized to entrust or receive entrustment in the field of business:
A. Asset Management
B. Insurance business;
C. Banking activities;
D. Buying and selling real estate.
Answer: EASY
Explanation: The Law on Credit Institutions, Article 106 only states that “Commercial banks are
entitled to entrust, receive entrustment, and act as agents in the fields related to banking activities,
insurance business, and asset management according to regulations. regulations of the State Bank”.
Verse 1 . Which of the following statements false:
a. Financial services include all banking and other financial services (exceptinsurance);
b. When the bank performs financial services, it will not change either side of the bank's balance
sheet;
c. Financial services help banks supplement profits and spread risks in business;
d. Financial services in banks are services associated with banking activities.
Question 2. In what way is the guarantee by the method of selling all or nothing different from the
other guarantee methods?
a. Commitment to buy all issued securities regardless of whether they are fullydistributed or not;
b. If the amount of securities sold reaches a ratio lower than the required rate, theentire issuance will
be canceled;
c. If the issued securities are not sold out, the entire issuance will be canceled andthe money sold will
be owned by the underwriting organization;
d. If the issued amount of securities is not sold out, the entire issuance will be canceled and the money
sold will be returned to the buyer.
Question 1. Which financial services do commercial banks not need to establish or acquire
subsidiaries or affiliates?
a. Underwriting securities;
b. Consulting services;
c. Insurance;
d. Financial leasing.
Question 2: Choose the TRUE statement:
a. In preservation and deposit services, the Bank still has to pay interest to customers if the preservation
is money;
b. Futures contracts are still traded on the OTC market;
c. An options contract does not oblige its buyer to execute the committed transaction, but an option
premium is required in all cases.
d. Large-scale financial leasing companies are still allowed to contribute capital, buy shares, establish
subsidiaries and associate companies.
| 1/44

Preview text:

lOMoARcPSD| 36667950
1. Which market do banks participate in to buy and sell short-term securities? a. Currency market . b. Capital market c. Forex market d. Gold market.
2. The service to pay money from customer's account to the address requested by the customer is:
a. Payment service using tax authorization
b. Payment service using periodic payment authorization c. Treasury Services
d. Account management services and safes
3. Which of the following is not consistent with deposit management principles ?
a. The bank will provide information about the husband's savings deposit balance when requested by the wife
b. The bank will not provide information about the husband's savings deposit balance at the request of
the wife (except for the couple who are co-owners).
c. The wife can use her own passbook as security for her husband's loan transaction at the bank.
d. All of the above sentences are correct
4. Banks are classified as financial intermediaries because...
a. Banks act as intermediaries in the markets
b. The main assets of a bank are financial assets
c. The operation of a bank consists of two stages: fund mobilization and fund use
d. Banks do not create their own claims, only transfer claims between the surplus and the deficit (số tiền thiếu hụt)
5. Which statement most accurately reflects the concept of market risk?
a. Fluctuations (sự biến động) in market price factors adversely (bất lợi) affect the value of assets of the bank
b. Fluctuations in market price factors adversely affect the value of the bank's assets and liabilities
c. Fluctuations in market price factors adversely affect the value of assets or liabilities of the bank.
d. Fluctuations in market price factors adversely affect the value of assets and liabilities of the bank.
6. Distinguish (phân biệt) between magnetic (từ tính) and electronic cards in:
a. Origin of the card => debit card, credit card and prepaid card
b. The type of card reader suitable for it c. Scope of card use
d. Immediate or deferred payment method
7. The minimum capital adequacy ratio (CAR coefficient) is calculated:
a. Own capital / Total assets at risk
b. Own capital/Total assets c. Equity/Total Assets
d. Equity/Total assets at risk
8. Which of the following statements is not correct?
a. Risks in bank credit are inevitable (không thể tránh khỏi) and cannot be completely eliminated (loại bỏ)
b. Full repayment of both principal and interest is the essence (bản chất) of bank credit
c. In the credit contract, there is no content that shows the customer's commitment to unconditionally
repay the bank when the debt is due. lOMoARcPSD| 36667950
d. Transactional assets in bank credit are diverse, which can be in the form of currency, real assets or signatures.
9. Which items are not included in the “Liabilities” of the bank: a. Common shares b. Term Deposit
c. Repurchase agreement (thỏa thuận mua lại) d. Bonds issued by banks 10.Payment by check will
a. Faster than paying by proxy
b. Slower than paying by ATM
c. All payment methods are the same
d. Not enough grounds (các căn cứ) to conclude the above ideas
11.In Vietnam today, the most influential factor on savings deposits of commercial banks is
a. Financial capacity (khả năng) of commercial banks b. Service quality of NH
c. Interest rate of capital mobilization of commercial banks
d. Trading network of commercial banks
12.Which of the following statements correctly reflects the theory of portfolio management by economist Markowitz?
a. Diversification in asset investment will help lower the risk of the whole portfolio than the risk of the
individual asset classes combined.
b. Diversification (đa dạng hóa) in asset investment will make the risk of the whole portfolio higher than
the risk of individual asset classes combined.
c. Specialization (chuyên môn hóa) in asset investments will help lower the risk of the entire portfolio
than the risk of the individual asset classes combined.
d. Specialization in asset investing will make the risk of the whole portfolio higher than the risk of the
individual asset classes combined.
13.Which of the following tools is considered an on-balance sheet tool to help banks reduce credit portfolio risk?
a. Debt secularization (chứng khoán hóa nợ)
b. Credit Risk Swap (hoán đổi rủi ro tín dụng) c. Debt trading d. None of the above tools
14.What is the minimum legal capital required by Basel I (1988)? a. 5% b. 6% c. 7% d. 8%
15.Which of the following valuable papers can be purchased and sold by commercial banks? a. Treasury Bills b. Government bonds c. Saving d. a and b
16.Which of the following is not true about Banker's Acceptance? lOMoARcPSD| 36667950 a. Not transferable
b. For a bank accepting a bill of exchange, agreeing to seal the acceptance is considered a form of
signature credit for the customer who is the issuer.
c. A bill of exchange issued by an enterprise
d. To be stamped “accepted” by the bank on the bill of exchange
17.Which of the following factors does not affect a bank's financial investment activities?
a. The development of financial markets
b. State investment incentives (ưu đãi) and support policies for hi-tech parks c. Bank's own capital d. Laws of the State
18.Which of the following services is not part of the treasury service of commercial banks?
a. Currency exchange at the request of the customer
b. Liquidation (thanh lý) of assets according to the wil l
c. Cash collection at customer's location d. Transfer money overseas
19.Which of the following statements is not true about derivatives?
a. The birth of derivatives stems from (bắt nguồn từ) the potential risk in contract transactions in the economy
b. Derivatives are always born before an underlying contract transaction
c. Derivatives are a form of hedging (phòng ngừa) financial risks when performing economic contracts
d. Using derivatives will help customers prevent and reduce financial risks
20.Which of the following types of contracts does not belong to derivatives? a. Futures contract b. Swap contract c. Credit agreement d. Option contract
1. In order to improve the capital adequacy ratio (CAR), commercial banks need to take the following measures: a. Increase charter capital b. Reduce risky assets
c. Increase in equity combined with a decrease in risky assets d. Increase risky assets
2. Which of the following loan and customer evaluation methods is a qualitative (định tính) method?
a. Method of artificial nervous system
b. Judgment (phán đoán) method c. Credit score method
d. All of the above are correct
3. When Commercial Bank A uses a ladder strategy (chiến lược bậc thang/ phân tầng) to invest in
securities in the financial market, if the maximum term for the portfolio is 10 years, the investment
amount will be divided into: a. 5 equal parts b. 10 equal parts c. 15 equal parts d. 20 equal parts
4. Which of the following statements is not correct? lOMoARcPSD| 36667950
a. Commercial banks are directly engaged (kinh doanh) in insurance business
b. Commercial banks are allowed to contribute capital and buy shares of enterprises operating in the insurance sector
c. Commercial banks are allowed to establish subsidiaries (công ty con) operating
in the field of collateral management
d. Commercial banks must establish subsidiaries to carry out insurance business activities
5. According to current regulations, a joint-venture commercial bank is a commercial bank established
in the form of a legal entity:
a. Limited company (including 1 member and 2 members or more)
b. Limited Liability Company and Joint Stock Company
c. Limited liability companies with two or more members and Joint stock companies
d. Limited liability company with two or more members 6. How is debt trading done? a. Participating in a loan b. Participating in time c. Transfer
d. All of the above are correct
7. An enterprise A in Ho Chi Minh City wants to pay for goods to a supplier in a northern province with
a value of VND 10 billion. Enterprises should use vehicles a. Travel Secrets b. Bank card
c. Accreditative (ủy nhiệm) d. commission
8. Which of the following statements is not true about derivatives?
a. The birth of derivatives stems from the potential risk in contract transactions in the economy
b. Derivatives are always born before an underlying contract transaction
c. Derivatives are a form of hedging financial risks when performing economic contracts
d. Using derivatives will help customers prevent and reduce financial risks
9. The trust matrix model (mô hình ma trận tin cậy) is considered a marketbased pricing model because:
a. The model of the machine complies with (phù hợp với) the laws of the market
b. The market price factor does not affect this model
c. This model uses the market probability distribution to determine default risk
d. This model not only determines the default risk event, but also determines the decrease in the market value of the loan 10.The CAMPARI rule is...
a. Method of evaluating customers through factors: Cash (Cash), capacity (Ability), loan interest
(Margin), loan purpose (Purpose), loan amount (Amount), repayment (Repayment) and loan guarantee
b. It is a quantitative assessment method
c. Both of these are true
d. Both of the above are wrong
11.Based on the card's technology, the bank's cards include: a. Magnetic and chip cards b. Magnetic and credit cards
c. Magnetic cards and ATM cards d. Chip cards and ATM cards lOMoARcPSD| 36667950
12.Which of the following statements is not correct?
a. In essence(về bản chất), interest rates are a type of price and are influenced by supply and demand
b. Basic interest rate is the highest interest rate that banks apply to lend to
reputable (uy tín) customers
c. Base interest rate is the floor of lending interest rate
d. The base rate indicates (cho biết) the minimum income on the bank's loan amount
13.The main uses of payment cards are: a. Cash withdrawals
b. Pay for goods and services c. Loan d. All of the above are true
14.Activities that are not financial investment activities of Vietnamese commercial banks include:
a. Contribute capital to buy shares of enterprises
b. Buying and holding shares of other credit institutions banks
c. Loans to business customers
d. Invest in the financial market
15.Corporate customers are not allowed to own any of the following deposit types a. Saved money b. Deposit payment c. Term Deposit
d. Certificates of deposit
16.Banks are classified as financial intermediaries because...
a. Banks act as intermediaries in the markets
b. The main assets of a bank are financial assets
c. The operation of a bank consists of two stages: fund mobilization and fund use
d. Banks do not create their own claims, only transfer claims between the surplus and the deficit
17.The Bank purchases investment securities with fixed maturities and holds them to maturity in order to:
a. Credit risk prevention
b. Hedging foreign exchange risk
c. Increase income from receiving interest at the end of the period
d. Price increases revenue from the difference between buying and selling prices
18.Which of the following services is not part of the treasury services of commercial banks?
a. Currency exchange at the request of the customer
b. Liquidation of assets according to the wil l
c. Cash collection at customer's location d. Transfer money overseas
19.Which of the following is not the purpose of securities investment activities for commercial banks?
a. Provide liquidity to the bank when needed
b. Regulating the stock market
c. Diversify profitable activities
d. Minimize the risk of concentration 20.NHM mainly works by:
a. Equity contributed by shareholders
b. Capital mobilized in the economy, in society c. Granted budget capital lOMoARcPSD| 36667950
d. Loans in the interbank market.
1. According to current regulations, domestic commercial banks are established and organized in the form of:
a. Joint Stock Company or Limited Liability Company
b. State-owned company or 1-member LLC c. JSC or Private Company
d. JSC or LLC 1 member
2. Choose the wrong sentence
a. Equity capital of commercial banks affects the reputation (uy tín) of the bank, the safety and
reputation of the bank is also an important factor in creating trust for customers, attracting a large amount of deposits.
b. The strict control of the Central Bank such as interest rates, exchange rate policy, the regulation of the
required reserve ratio indirectly affects the capital mobilization of commercial banks.
c. Loans from other credit institutions and loans from the State Bank usually account for a large
proportion of the total business capital of commercial banks
d. The charter capital (vốn điều lệ) of the bank must be at least equal to the legal capital (vốn pháp định)
3. Please choose the most correct statement
a. Current deposits usually have higher interest rates than demand deposits
b. Current deposits have less volatile (dễ thay đổi, biến động) balances than demand deposits
c. Paying deposit has a credit balance, it can also have a debit balance
d. Demand savings deposit with outstanding balance
4. According to ......., what types of banks include commercial banks, commercial banks, cooperative banks? a. Operational goals b. Business method c. Banking d. Area of operation
5. Disadvantages of one-time payment via SBV a. Slow payment speed
b. Only applicable to banks in the same system
c. Signature samples must be registered at the lead bank and other member banks d. High cost of use
6. To hedge against the risk of exchange rate decline at the time of receipt of foreign currency in the future, the exporter can:
a. Exercising (thực hiện) the right to buy foreign currency today
b. Exercising a foreign currency put option today
c. Sell foreign currency call options today
d. Exercising to sell foreign currency put options today
7. The bank has purchased a number of corporate bonds, in which the repurchase agreement was
agreed upon when issued. The issuer will buy back these bonds when a. Market interest rates rise b. Market interest rates fall
c. Market interest rates fall, bond prices increase by the redemption price and reach the agreed maturity d. Bank in trouble
8. According to current regulations, which of the following statements is not true? lOMoARcPSD| 36667950
a. Credit institution is an enterprise that carries out one, several or all banking activities
b. Credit institution is a commercial bank that performs one, several or all banking activities
c. Commercial bank is a type of bank that can carry out all banking activities
d. Bank is a type of credit institution that can carry out banking activities.
9. Which of the following is an on-balance sheet transaction of a bank?
a. Buy corporate bonds b. Advise
c. Investment Management d. Insurance
10.Deposits in trust at ... not insured a. Securities Company b. NH
c. People's Credit Fund
d. Microfinance institutions
11.In which areas are the subsidiaries (công ty con) of commercial banks allowed to operate?
a. Secured asset management (quản lý tài sản đảm bảo)
b. Forex trading (kinh doanh ngoại hối)
c. Factoring (bao thanh toán)
d. All of the above statements are correct
12.The mechanism (cơ chế) of action of credit risk swaps is similar to a. Credit insurance b. Debt trading form
c. Securitization (chứng khoán hóa) d. All is incorrect
13.If the bank expects the spot rate in 3 months to be lower than the 3 month forward rate today,
how will the bank speculate on the price?
a. Buy foreign currency for a term of 3 months, at the end of the drought period, it will be sold for immediate delivery
b. Selling foreign currency for a term of 3 months, at the end of the term will buy for immediate delivery
c. Buy foreign currency with term of 3 months, at the end of term will buy spot
d. Selling foreign currency for a term of 3 months, at the end of the term will sell for immediate delivery
14.Which of the following statements is not related to concentration risk (rủi ro tập trung)?
a. Ratio of using short-term sources for medium and long-term loans does not exceed 30%
b. For 1 customer, no more than 15% of the bank's own capital
c. The ratio of outstanding loans to non-production loans does not exceed 16% of total outstanding loans
d. The proportion of loans for securities business does not exceed 20% of the bank's charter capital
15.Which of the following is an indirect credit transaction?
a. Loan under overdraft (thấu chi) limit
b. Discount negotiable c. Financial leasing
d. Lending each time (cho vay từng lần)
16.Which asset class is highly liquid and profitable? a. Gold, foreign currency b. Corporate bonds
c. Business capital contribution d. Government bonds lOMoARcPSD| 36667950
17.What measures should commercial banks not apply when short-term liquidity surplus?
a. Interbank deposit lending b. Borrow on interbank
c. Invest in short-term valuable papers
d. Both b and c are correct
18.Which of the following is a qualitative assessment method for customers and loans?
a. Method of artificial nervous system b. Judgment method c. Credit score method
d. All of the above are correct
19.Attracting deposits by technical means including
a. Arrangement of ATMs at colleges and universities
b. Issuing multi-purpose cards, diversifying customers, providing white check services for customers
c. Issuing multi-purpose cards, giving raincoats to customers, saving money, diversifying deposit methods
d. Issuing ATM cards at colleges and universities, providing blank check services to customers, increasing the number of bank staff.
20.The Credit Scoring system is used to a. Credit officer management
b. Customer analysis in deposit transactions
c. Improve the quality of payment by credit card
d. Slap all of the above ideas wrong.
1. Similarities in check payment method and payment order payment method
a. Payment certificate issued by the debtor
b. Chances of encountering fake vouchers
c. Is transferred directly in the delivery d. All right
2. When we say securities with high liquidity, we mean
a. Such securities are readily convertible (có thể chuyển đổi) to cash
b. Such securities are difficult to resell (bán lại) in the market
c. Difficult to convert such securities into cash
d. Stocks with high prices
3. Which source of capital is the capital source to pay interest of commercial banks?
a. Deposits and loans to the Treasury, the State Bank and other credit institutions b. Other Receivables
c. Deposits from customers and issuance of valuable papers d. Both a and c
4. Distinguish between electronic cards and electromagnetic cards in: a. Origin of the card
b. The type of card reader suitable for it c. Scope of card use
d. Immediate or deferred payment method
5. The credit portfolio risk measurement model helps banks:
a. Find a group of customers with absolutely no risk
b. Assess the working ability of credit officers
c. Increase credit scale without fear of any risk
d. All of the above are incorrect lOMoARcPSD| 36667950
6. What is the maximum capital contribution ratio of a bank compared to the value of its own capital and reserves. a. 30% b. 40% c. 50% d. 60%
7. Which item does not belong to the profitable assets of commercial banks.
a. Term deposits of domestic and foreign credit institutions
b. Term deposits with economic and residential institutions c. Investment in securities
d. Residential loan (cho vay mua nhà)
8. Money management service on account.........
a. Coupled with deposits, it should be an on-balance sheet transaction
b. Usually only applied to individual customers with long-term deposits
c. Usually applied to customers with large deposit balances
d. It is an unprofitable business because it is off-balance sheet
9. The KMV model is often used to. a. Invest in gold b. Stock investment c. Invest in bonds
d. Foreign currency investment
10.A bill of exchange accepted by a bank is considered a highly secure instrument because:
a. The bank is first liable, having to pay the entire debt it secures to the beneficiary.
b. Issued and guaranteed by the government c. Low interest rates d. Absolutely no risk
11.Which of the following is not included when calculating loan interest? a. Level of default risk b. Tenor (kỳ hạn)
c. C cost of deposit mobilization d. Cost rent
12.In the entrusting business:
a. The entrusting party transfers the property rights to the trustee under the contract
b. The trustee transfers the entire and permanent property rights to the trustee
c. The trustee operates for the benefit of the beneficiary d. Both a and c are correct
13.Which of the following statements is correct?
a. Due to the phenomenon (hiện tượng) of information asymmetry (bất cân xứng), the bank has the
ability to accept a bad customer at the same time as rejecting a good customer.
b. Due to the phenomenon of information asymmetry, the bank is able to accept a good customer while
at the same time rejecting a bad customer.
c. Both of the above are correct
d. Both of the above are wrong
14.Corporate customers may not own any of the following: a. Saved money b. Deposit payment c. Term deposit lOMoARcPSD| 36667950
d. Certificates of deposit
15. In order to improve the capital adequacy ratio (CAR) – hệ số an toàn vốn,commercial banks need
to take measures. a. Increase charter capital b. Reduce risk assets
c. Increase in equity combined with (kết hợp với) a decrease in risky assets d. Increase risky assets
16. Transactional and non-trading deposits are distinguished from each other oqr a. Customers send money b. Deposit term
c. Purpose of sending money
d. Sentences b and c are correct
17. Property guardian (giám hộ) service
a. As guardian of the client's previous activity
b. Asset management in case the heir is a minor (chưa thành niên) c. Manage all customer needs d. All three factors above
18. Interest rate risk occurs when
a. Fluctuating market interest rates affect the bank's net interest income and capital value.
b . Fluctuating market interest rates adversely affect a bank's net interest income and/or capital value
c. Rising market interest rates affect the bank's net interest income and capitalvalue
d. Falling market interest rates affect the bank's net interest income and capitalvalue
19. According to current regulations, a joint-venture commercial bank is acommercial bank
established with the form of a legal entity:
a. Limited Liability Company (including 1TV AND 2TV or more)
b. Company Limited and Joint Stock Company
c. Limited liability companies with 2 or more members and JSCs
d. Limited liability company with 2 or more members
20. Which of the following factors is not necessary to be negotiated (được đàm phán) in the credit
risk swap contract? a. Reference property b. Credit risk events c. Payment methods
d. Exchange rate changes in the market
1. In Vietnam, the operation of a financial institution is similar to that of an investment bank: a. Insurance company
b. Financial leasing companies c. Commercial Bank d. Securities Company
2. Which of the following methods represents a bank's credit operations? a. Transfers
b. Thanks to the smooth collection
c. Thanks for collecting vouchers d. Issuance of L/C
3. Which of the following operations cannot be grouped with the rest: a. Personal loan b. Bank guarantee c. Other bank loans lOMoARcPSD| 36667950 d. Discount valuable vouchers
4. Arrange the following sentences in the correct order of payment:
1. Buyer and seller have a relationship to buy and sell goods and/or services with each other
2. Buyer's bank pays the seller through the seller's bank
3. The buyer writes a payment order to ask his bank to pay for the purchase to the seller
4. The seller's bank notifies (thông báo) the seller that the buyer has paid a. 1, 2, 3, 4 b. 1, 3, 4, 2 c. 1, 3, 2, 4 d. 1, 2, 4, 3
5. The objective of the trust matrix model is to:
a. Building a business strategy for the bank
b. Completing the bank's credit policy
c. Use calculation methods to measure the minimum risk exposure of each loan as well as the entire portfolio
d. Assess (đánh giá) the quality of the bank's investment
6. Which of the following statements is not true about the operation of the NHS? a. Provide financial advice
b. Issuing certificates of deposit, promissory notes, bills, and bonds to raise capital
c. Using mobilized capital to contribute capital, buy shares
d. Open an account to pay at another credit institution 7. Bancassurance is:
a. The combination of money transfer and insurance business
b. Combination of deposit and insurance operations
c. Combination of credit and insurance operations
d. Combination of banking and insurance
8. What activities does the commercial bank directly perform:
a. Underwriting of securities issuance b. Insurance Business c. Financial leasing d. All is incorrect
9. Which sentence is not the same between payment by payment order and collection order?
a. There are 2 forms of payment through payment intermediaries
b. Orders made by the account holder must be made on pre-printed forms provided by the payment service provider
c. Is a form of payment that the account holder makes an order, authorizes (ủy nhiệm) payment with
payment service providers to make payment.
d. Only used to pay different values, different periodicity (chu kỳ)
10. Question: Credit institutions contribute capital or buy shares of enterprises by:
a. Charter capital and reserve fund
b. Charter capital and reserve fund, mobilized capital
c. Charter capital and mobilized capital
d. Charter capital and reserve fund, and borrowed capital
11. The basis for determining the floating interest rate for loans is:
a. Basic interest rate announced by the State Bank plus expected inflation lOMoARcPSD| 36667950
b. Savings deposit interest rate plus a certain margin
c. Real interest rate plus announced interest rate d. All is incorrect
12. The main assets on a commercial bank's balance sheet are: a. Deposits
b. Workhouse and office equipment c. Loans to customers d. Guarantees
13. Which of the following statements is false?
a. Anonymous sec (séc ẩn danh) is a type of sec that does not include the beneficiary's name; and
anonymous checks are only transferable by endorsement
b. Identifier Sec is a type of sec specifying the beneficiary's name; sec identifier is not transferable without endorsement
c. Transfer Sec is not transferable and cannot be cashed
d. Cash Sec is paid by the paying bank in cash
14. Which of the following tools cannot be used to reduce credit portfolio risk? a. Credit risk swaps b. Option contract c. Futures contract
d. b and c are both correct 15. How is debt trading done? a. Participating in a loan b. Participating deadlines c. Transfer d. All right
16. Banking operations related to commercial paper include: a. Collect commercial paper b. Discount negotiable
c. Storm received commercial paper d. All right
17. The meaning of securities investment activities for commercial banks is to:
a. Provide liquidity when needed
b. Diversify profitable activities c. Increase profits
d. The sentences above are all correct .
18. The form of payment is made between commercial banks of different systems and locations, but each
commercial bank that opens a deposit account at the other bank is a. Inter-bank payment b. Payment via SBV c. Clearing
d. Payment via agent bank relationship
19. If the customer's equity level in the plan is too low, this is related to... a. Choice risk b. Business risk c. Guaranteed risk lOMoARcPSD| 36667950 d. Intrinsic risk
20. The minimum capital adequacy ratio (CAR) reflects the relationship between
a. Own capital and risky assets of commercial banks
b. Own capital and converted risk assets of commercial banks
c. Own capital and assets of commercial banks
d. Own capital and credit balance of commercial banks.
Which asset class is highly liquid and profitable: a. Gold, foreign currency; b. Corporate bonds;
c. Contributing business capital; d. Government bonds;
1. Streamlining the portfolio management process:
1. Consulting on investment types 2. Define Expectations
3. Sign investment portfolio management contract
4. Build the optimal investment structure. 5. Provide information 6. End of contract a. 1,2,3,4,5,6; b. 3,5,1,2,4,6; c. 5,3,1,2,4,6; d. 1,5,3,2,4,6;
2. Significance of securities investment activities for commercial banks:
a. Customer service development; b. Brand enhancement;
c. Provide liquidity when needed;
d. Minimizing investment risks;
3. According to current regulations, for the performance of securities brokerage activities, commercial banks… a. Absolutely not allowed ;
b. Unlimited exercise is allowed ;
c. Direct execution is not allowed ;
d. Permitted through the Stock Exchange ;
4. Which of the following statements is not true about derivatives?
a. The advent of derivatives stems from the potential risks in contract transactions in the economy ;
b. Derivatives are always born before an underlying contract transaction ;
c. Derivatives are a type of financial hedging when performing economic contracts ;
d. Using derivatives will help customers prevent and reduce financial risks ;
5. Which of the following types of contracts does not belong to derivatives? a. Futures contract ; b. Swap contract ; c. Credit contract ; d. Option contract ;
6. Which of the following is not related to portfolio management? a. Define expectations ;
b. Building the optimal (tối ưu) investment structure ;
c. Consulting on investment types ; lOMoARcPSD| 36667950 d. Securities investment ;
7. Credit institutions contribute capital to purchase shares of enterprises by:
a. Charter capital and reserve fund;
b. Charter capital and reserve fund, mobilized capital;
c. Charter capital and mobilized capital;
d. Charter capital and reserve fund, borrowed capital;
8. Which function is not of securities investment activities:
a. Create geographical diversification;
b. Strengthen the capacity of enterprises to realize business opportunities;
c. Mitigating (giảm thiểu) the impact of taxes; d. Balance credit risk ;
9. Which is not the meaning of securities investment activities for commercial banks:
a. Provide liquidity when needed; b. Acquisition of businesses;
c. Diversify profitable activities;
d. Reduce the risk of concentration;
10. When commercial bank A uses a ladder strategy to invest in securities in the financial market, if the
maximum maturity for the portfolio is 10 years, the investment amount will be divided into: a. 5 equal parts ; b. 10 equal parts ; c. 15 equal parts ; d. 20 equal parts ;
11.Which of the following statements is not true about the ladder strategy in financial investment activities of banks?
a. The bank is easy to manage, just allocate (phân bổ) investment evenly for the terms and regularly
check the due accounts for reinvestment ;
b. Limit fluctuations in the portfolio's income, by maintaining an average return, and avoiding major loss events ;
c. Banks need to forecast future interest rates ;
d. All of the above are correct ;
12.Choose the best statement about the term division strategy (A spit-maturity approach)?
a. Fully invest in short-term securities ;
b. Fully invest in long-term securities ;
c. Investing in short-term and long-term securities ;
d. The term division strategy includes all three sub-strategies mentioned above ;
13. Banks use Barbell strategy in stock investment, ire:
a. Portfolio with only short-term securities ;
b. The portfolio contains only long-term securities ;
c. Portfolio with a combination of short-term and long-term securities ;
d. All of the above are correct ;
14.Which of the following statements is not true about the term division strategy?
a. With this strategy, the bank can get larger profits than the ladder strategy ;
b. This strategy is considered an active investment strategy ;
c. Banks do not need to predict the direction of interest rate movements in theeconomy ;
d. The bank may incur a loss if the bank's predictions are inaccurate ; lOMoARcPSD| 36667950
15.Which of the following statements is not true about derivatives?
a. The advent of derivatives stems from the potential risks in contract transactions in the economy ;
b. Derivatives are always born before an underlying contract transaction ;
c. Derivatives are a type of financial hedging when performing economic contracts ;
d. Using derivatives will help customers prevent and reduce financial risks ;
16. Which of the following types of contracts does not belong to derivatives? a. Futures contract ; b. Swap contract ; c. Credit contract ; d. Option contract ;
17. Which of the following is not related to portfolio management? a. Define expectations ;
b. Building the optimal investment structure ;
c. Consulting on investment types ; d. Securities investment ;
18.The KMV model is often used to: a. Gold investment; b. Stock investment; c. Bond investment;
d. Foreign currency investment; 19.Secularization:
a. Is the issuance of securities on the basis of the value of playable that a bank already owns;
b. Is the issuance of bonds on the basis of the value of playable that a bank already owns;
c. Is the issuance of securities on the basis of the value of receivables that a bank already owns;
d. Is the issue of shares on the basis of the value of the receivables that a bank already owns;
20.The bank has purchased a number of corporate bonds, in which the repurchase agreement was agreed
upon when issued. The issuer will redeem these bonds when:
a. Market interest rates increase;
b. Market interest rates fall;
c. The market interest rate decreases, the bond market price increases by the
redemption price and the agreed maturity; d. Banks in trouble;
21.Which of the following is true?
a. Treasury bills are government debt certificates issued by the State Bank;
b. Treasury bills are government debt certificates issued by the State Treasury;
c. Treasury bills are not transferable on the money market;
d. All of the above are correct;
22.The Bank purchases investment securities with fixed maturities and holds them to maturity in order to: a. Credit risk prevention ;
b. Hedging foreign exchange risk ;
c. Increase income from receiving interest at the end of the period ;
d. Increase revenue from the difference between buying and selling prices ;
23.If a commercial bank wants to distribute (phân phối lại) securities investment fund certificates:
a. An investment fund must be established under ;
b. Must establish a securities brokerage company ;
c. Must acquire securities portfolio implementation (thực hiện) company ;
d. All of the above are correct ; lOMoARcPSD| 36667950
24.Which of the following statements is most correct?
a. Government bonds have the lowest credit risk and the highest return ;
b. Government bonds have the lowest credit risk but at the same time have low profitability ;
c. Corporate bonds also have low credit risk ;
d. All of the above are correct ;
25.Commercial banks are allowed to contribute capital or purchase shares of enterprises operating in
which of the following fields? a. Insurance ; b. Securities ; c. Credit information ;
d. All of the above are correct ;
26.What kind of valuable papers are not issued by the enterprise:
a. Commercial deed (chứng thư thương mại)
b. Bill of exchange accepted by the bank ; c. Certificate of Deposit ; d. Corporate bonds ;
27.A bank's portfolio should be designed so that:
a. Guarantee the lowest cost ;
b. Ensure profits at a certain level of expectations but risks must be at a minimum ;
c. Highest return and acceptable risk ; d. b, c are both correct ;
28.The bank used mobilized sources to buy bonds with a fixed rate of return, while deposit interest rates
increased according to the market. How will this affect the bank? a. Risk of default ; b. Interest rate risk ; c. Credit risk ; d. Completely unaffected ;
29.Which of the following is not the purpose of securities investment activities for commercial banks?
a. Provide liquidity to the bank when needed ;
b. Regulating (điều tiết) the stock market ;
c. Diversify profitable activities ;
d. Reduce the risk of concentration ;
30.Banks invest in fixed-yield securities such as government bonds. How will an increase in market interest
rates affect the market price of previously issued debt securities?
a. Increases the market price of debt securities ;
b. Decreasing the market price of debt securities ;
c. Does not change the market price of the debt securities ;
d. All of the above are wrong ;
31. Banks invest in fixed-yield securities such as government bonds. How will an increase in market
interest rates affect the market price of previously issued debt securities?
a. Increases the market price of debt securities ;
b. Decreasing the market price of debt securities ;
c. Does not change the market price of the debt securities ;
d. All of the above are wrong ;
32. Which of the following factors does not affect a bank's financial investment activities?
a. The development of financial markets; lOMoARcPSD| 36667950
b. The State's preferential policies and investment support for hi-tech parks; c. The bank's own capital; d. Laws of the State;
33. The bank's portfolio will focus on securities such as government bonds, Treasury bills, because:
a. These types of securities are not transferable on the market ;
b. These securities have low credit risk and are easily transferable in the market to supplement liquidity when needed ;
c. These types of securities have higher profitability than others ;
d. All of the above are correct ; 34.
Financial investment activities help banks:
a. Increase the balance of term between capital sources and use of capital ; b. Limit interest rate risk ; c. Dispersing credit risks ;
d. All of the above are correct ; 35.
In some cases, a bank's portfolio is not for profit, but for: a. Use idle money; b. Create liquidity; c. Increased riskiness;
d. Increasing the bank's competitiveness;
36. Which of the following investment securities is the least safe? a. Treasury Bonds; b. Treasury Bills;
c. Banker's Acceptance of Bill of Exchange; d. Corporate bonds; 37.
When investing in securities, credit risk occurs when:
a. Market interest rates fluctuate;
b. The commercial bank has not used up its credit limit;
c. Investors massively sold securities;
d. The issuer of securities is unable to pay the principal and interest; 38.
Why do investors prefer issuer callable bonds? a. Stable income;
b. Not affected by fluctuations in the market;
c. These securities have higher yields than others; d. High degree of safety; 39.
REPO transactions include: a.Selling securities b. Buyback of securities c.Only b is correct skin and b 40.
When you say securities are highly liquid, what do you mean?
a. Such securities are readily convertible to cash;
b. Such securities are difficult to resold in the market;
c. Difficulty converting such securities into cash;
d. Securities with high selling price;
41. Vietnamese commercial banks performing securities underwriting operations:
a. It is not necessary to have the approval of the State Bank;
b. The Law on Credit Institutions 2010 does not allow commercial banks to carry outsecurities underwriting operations;
c. Must establish a securities company (subsidiary of the Bank); lOMoARcPSD| 36667950
d. Must be done at the Stock Exchange;
42. The influence of political and economic events causing a sharp drop in stock prices on the stock market belongs to: a. Interest rate risk; b. Credit risk; c. Market risk; d. Risk of acquisition;
43. Which market do banks participate in to buy and sell short-term securities? a. Money market ; b. Capital market ; c. Foreign exchange market ; d. Gold market ;
44. Activities that are not financial investment activities of Vietnamese commercial banks include:
a. Contribute capital to buy shares of enterprises
b. Buying and holding stocks of other credit institutions/banks
c. Loans to business customers
d. Invest in the financial market
45. Which of the following valuable papers can be purchased and sold by commercial banks? a. Treasury bills ; b. Government bonds ; c. Savings book ; d. a and b ; 46.
What kind of valuable paper is considered a certificate of debt? a. Commercial paper ; b. Company shares ; c. Dividends ;
d. All of the above are correct ; 47.
Investment activities help banks improve liquidity by:
a. Convert financial assets into cash according to the fourth priority: Highly liquidassets will be sold later ;
b. Convert financial assets into cash in order of priority: Assets with low liquidity aresold first ;
c. Convert financial assets into cash in order of priority: Liquid assets are sold first ; d. All of the above are correct ; 48.
Treasury bills are issued to:
a. Raising capital for the company ;
b. Serving the company's investment and development needs ;
c. To make up for the shortfall of the State Budget ;
d. Increase spending sources for the company ; 49.
Which of the following is not true about corporate bonds?
a. As a type of debt security ;
b. Issued to finance local infrastructure projects ;
c. Confirm the debt repayment obligations of the issuer towards bondholders ;
d. Is one of the valuable papers that the Law allows banks to participate in themarket ;
50. What are the basic similarities between treasury bills and treasury bonds? a. Release period ; b. Release purpose ; c. Issuing organization ;
d. All three of the above comments are correct ;
51. The joint venture capital of commercial banks is deducted from?
a. Charter capital of the bank ;
b. Deposits mobilized from customers ; lOMoARcPSD| 36667950 c. Bank's reserve fund ; d. a and c ;
52. What is the method of securities underwriting in which the underwriter will commit to underwriting
all of the issued securities whether or not they are fully distributed?
a. Guarantee with commitment to try the best ;
b. Guarantee with firm commitment ;
c. Guarantee according to the minimum - maximum method ;
d. All of the above are wrong ; 53.
What is not true about treasury bills?
a. Government debt certificates issued by the State Treasury ;
b. Issued by a private company ;
c. Instruments may carry no interest or low interest rates ;
d. Issued to make up for the deficit of the State Budget and realize the objectives of the monetary policy ;
54. Commercial banks participate in the money market in which of the following forms?
a. Participating in Treasury bill bidding
b. Buying and selling negotiable instruments, Government bonds c. Buy State Bank bills
d. All of the above are correct;
55. A bill of exchange accepted by a bank is considered a highly secure instrument because:
a. The bank is first liable, having to pay the entire debt it secures to the beneficiary.
b. Issued and guaranteed by the Government c. Low interest rates d. Absolutely no risk Part 2
1. Which method shows the bank's credit operations? a. Transfers;
b. Thanks to the smooth collector;
c. Thanks to the collection of documents; d. Issuing letters of credit;
2. What are the current forms of payment via bank?
a) Clearing, payment under contract, UNC, check;
b) Plan payment, transfer payment, cash payment;
c) Payment through multiple banks, payment through one bank;
d) Checks, payment orders, collection orders, letters of credit, card payments. 3.
The payment range of Wire Transfer Checks is:
a) Within the national economy: between businesses, between businesses and individuals, between individuals and businesses;
b) Within the scope of commercial banks in the same system, other commercial banks in the system,
commercial banks with the State bank, the State bank with commercial banks and bank branches with each other;
c) Among customers who have opened accounts at the same or different banks but these banks
participate in clearing in the province or city;
d) Within the range between customers who open accounts at the same bank.
4. Which of the following accounts are not used in the Interbank Payment System:
a) Payment deposit accounts of members; b) Clearing account;
c) Collection and payment accounts;
d) Customer's savings account. lOMoARcPSD| 36667950 5.
Places where checks can be paid:
a) At the place of business of the drawee;
b) At the place indicated on the check;
c) At any credit institution, the bank is allowed to issue checks; d) All 3 statements are true.
6. Which of the following is not a form of payment between banks: a) Payment via State bank; b) Deferred payment; c) Clearing;
d) Payment through correspondent banking relationship.
7. : Which is not the advantage of clearing between banks: a)
Fast, can handle multiple payment transactions at the same time; b)
Creating conditions for banks to effectively manage capital and harmonize capital; c) Property safety; d)
Clearly show the receivable and payable amount of each bank participating in clearing.
8. : If the amount written in words and numbers on the face of the promissory note do not match, the
payment will be based on which amount: a) Amount in words; b) Amount in number;
c) The smaller of the 2 numbers;
d) That bill of exchange has no value.
9. Delays in issuing a spending authorization can harm: a) Buyer; b) Seller; c) Buyer and seller;
d) The bank provides payment services to buyers.
10.Which of the following is not true about payment methods in the form of collection orders:
a) Can be used in case the two parties buy - sell have mutual trust, have agreed on payment conditions;
b) Due to the beneficiary making the payment process may be delayed;
c) As established by the beneficiary, the payment process does not occur without delay;
d) The beneficiary's bank notifies the buyer through the buyer's bank.
11.: In what cases can the payment method by collection order be used?
a) In case two service providers do not have mutual trust;
b) In cases where the two parties buying and selling goods or providing services do not have mutual
trust or in the case of payment for services provided, there are accurate measuring devices such as
electricity, water, and telephone;
c) In case the two parties purchase and sell goods with mutual trust;
d) In cases where two parties purchase and sell goods or provide services with mutual trust or in the
case of payment for services provided, there are accurate measuring devices such as electricity, water, and telephone;
12.Which of the following statements is not true about the characteristics of the clearing method between banks? a) Wide payment range;
b) The member banks must open a deposit account at the lead bank;
c) Can process multiple payment transactions at the same time;
d) The lead bank is the State Bank or another banking unit if there is no branch of the State Bank in the locality. 13.
Credit cards have the following advantages:
a. Low transaction fees and narrow operating range. lOMoARcPSD| 36667950
b. Popular for car buying and selling activities.
c. Use payment as an alternative to cash but not a substitute for checks.
d. The card can be used to withdraw cash from a bank branch, correspondent bank or ATM or transfer to another bank's account;
14. which of the following is incorrect. When paying a check the customer must: a. Have an account at that bank; b.
Payment can only be made within the same bank system; c.
Have a serial number in the bank's serial number; d.
Write the amount on the check to be less than the account balance;
15. Which is not the same thing between the two check and card payment methods? a. All issued by the bank; b.
As a form of non-cash payment; c.
The payee can receive cash or wire transfer; d.
It is a form of payment through a bank intermediary. 16.
What is the difference between UNC and UNT? a.
The beneficiary and the payer must have an account at the bank; b. All created by the customer; c.
Service fee for money transfer paid by the beneficiary or the payer; d.
Make payments on UNC, UHT forms of NH. 17.
Which form of payment is the supplier of goods and services more
guaranteed to pay?
a. Sec ; b. Authorization for collection; c. Accreditative; d. Letter of credit. 18. Credit card features: a.
Any customer can have a credit card; b.
When using a credit card, the debt balance decreases gradually; c.
The cardholder needs to have a balance on his payment account at the card issuing bank; d.
A credit card has the same features as a debit card. 19. Please choose False:
Members participating in the NIEPS must be payment service providers and comply with the following conditions: a.
Must have a payment deposit account at the Exchange; b.
Must maintain the balance of the payment deposit account to secure the execution of
Payment Orders and clearing through the NIEPS; c.
Must establish, maintain and manage net debt limit in case of low value payment service; d.
With the consent of the direct management member. 20.
Why distinguish between the presentation period and the payment validity period: a.
To encourage the beneficiary to present the check promptly; b.
To encourage the use of check payments; c.
Extend the payment time, increase the validity period for the check, in case of unexpected cases; d.
Grounds for considering the drawer's violation of payment discipline; 21.
: What types of checks require the beneficiary to have a bank account a.
Crossed checks, wire transfers; b.
Bank transfer checks, travelers checks; c.
Wire transfer check, order check; d.
Traveler's check, slash check. lOMoARcPSD| 36667950 22.
Choose the correct answer: a.
The payment order must be made and signed by the payer and only based on that order for
the bank to pay money to the beneficiary; b.
The payment conditions of the payment order are that the beneficiary and the paying unit must have a bank account; c.
Debit cards have functions such as: direct payment, bank loan for payment, automatic withdrawal; d.
The credit card has functions such as: automatic withdrawal, direct payment, can carry the
balance over many consecutive months. 23.
Choose the correct answer: a.
ATM card used for payment and withdrawal; b.
Debit cards are for withdrawals only; c. Credit card for payment; d.
Payment card used to withdraw money. 24.
Interbank payments are made between which of the following banks? a.
Agribank district 9 branch and Vietinbank Binh Duong branch; b.
Vietcombank branch in district 9 and Vietinbank branch in district 9; c.
Agribank district 9 branch and Agribank Binh Duong branch; d.
Vietcombank branch in district 9 and Agribank branch in Binh Duong district;25. Which of the
following statements is correct?

a. To send money through another bank, the depositor must have an account at the bank;
b. To organize payment, 2 banks need each other's deposit accounts;
c. The main form of payment of the two banks in the same system is interbank payment;
d. Between two banks, the payment organization through correspondent bankingrelationship does
not need an additional bank as an intermediary; 26.
Select the correct answers: a.
Minimum balance of debit card is 0; b.
Credit cardholders who pay late payments will incur high interest rates; c.
The supplementary card is issued when the main card is lost; d.
Credit cards cannot withdraw cash. 27.Choose the wrong sentence: a.
An anonymous check holder is the legal beneficiary; b.
Wire checks are non-transferable and non-cashable; c.
It is possible to transfer part of the amount stated on the check; d.
The drawer must still pay the amount stated on the check after the check is rejected by the
payee under the notice of suspension of payment;
28. Which of the following is not true about payment by letter of credit? a.
Buyers save time and effort in finding reputable and reliable partners; b.
The payment process is short, the parties proceed very quickly in making and checking documents; c.
Usually applied to import and export payment activities; d.
Protect the interests of both buyers and sellers of goods when they do not have the conditions to
directly exchange with each other.
29. Disadvantages of payment through correspondent banking relationship: a.
Does not facilitate money transfers, it will send payment orders settled through an organized agent account with each other; b.
The payment involves 4 parties, the transaction step by step is not convenient; c.
Only within the banks of the same system, customers are not many; d.
Requires a modern and sensitive technology system when transacting between banks to ensure low risk;
30.Using which of the following cards will increase your debit balance when buying goods and services: a. Debit card; lOMoARcPSD| 36667950 b. Credit; c. Payment card; d. ATM card.
31.: Select the correct answers: a.
Payment cards are also known as debit cards; b.
The credit card only allows the account holder to withdraw funds within an agreed limit; c.
Checks are only used to withdraw cash at the bank; d.
Using a debit card means you are borrowing money from a bank.
32. Choose the wrong sentence, the customer wants to use the means of payment via the bank, the customer must have: a. Payment account;
b. Transaction deposit account; c. Demand savings account; d. Current account; 33.
:The payment card has no embossed information? a. The name of the card; b. Card number; c. Cardholder's ID number; d. Cardholder's name;
34.A customer uses a card to buy goods at a supermarket, the value of goods is immediately debited to
the cardholder's payment account and credited to the supermarket's account. The card type and payment method are:
a. Credit cards and electronic payment methods;
b. Debit cards and mechanical payment methods;
c. Credit cards and mechanical payment methods;
d. Debit cards and electronic payment methods;
35.Magnetic cards and electronic cards are distinguished from each other in:
a. Origin of the amount on the card; b. Scope of card use;
c. Techniques for reading and storing information on cards; d. Card payment method;
36.A customer uses a card inserted into the machine, after entering his personal identification number,
the customer withdraws an amount and his account is debited . What kind of card is it: a. Card from; b. Payment card; c. ATM card; d. Smart card;
37. Which of the following are characteristics of electronic payment methods:
a. Cardholders get deferred payment
b. The merchant's account will be credited after a certain number of days;
c. Used in conjunction with an electronic card;
d. There is no separation between transaction time and payment time; 38. When using a credit card, the cardholder will have to:
a. Deposit an amount on the card account;
b. Bear interest if payment is not enough within a certain number of days;
c. Have a payment deposit account at a bank;
d. Payment immediately upon receipt of periodic statement from the bank;
39. Authorization for collection and authorization for expenditure are similar in that:
a. As a means of payment via bank issued by customers;
b. The document circulation process is similar;
c. There may be a risk of underpayment upon presentation of payment documents; lOMoARcPSD| 36667950
d. The drawer is the purchaser;
40. A customer uses a card to buy goods at a supermarket, the value of goods is immediately debited
to the cardholder's payment account and credited to the supermarket's account. The card type and payment method are:
e. Credit cards and electronic payment methods;
Debit cards and mechanical payment methods;
Credit cards and mechanical payment methods; Debit cards and electronic payment methods; 41. Check means:
a. Order slip issued by the purchaser;
b. Bill of exchange issued by the seller;
c. Debit note issued by the bank;
d. Special draft issued by the account holder;
42. Which of the following are characteristics of electronic payment methods:
e. Cardholders are entitled to deferred payment;
f. The merchant's account will be credited after a certain number of days;
g. Used in conjunction with an electronic card;
h. There is a network connection between payment participants;
43. Magnetic cards and electronic cards are distinguished from each other in:
a. Origin of the amount on the card; b. Scope of card use;
c. Techniques for reading and storing information on cards; d. Card payment method; 44.
When using a credit card, the cardholder will have to:
a. Deposit an amount on the card account;
b. Bear interest if the minimum payment is not met after the specified time limit;
c. Have a payment deposit account at the bank;
d. Pay immediately upon receipt of periodic statement from the bank;
45. According to the Law on Transferable Instruments, if the time of force majeure or objective obstacles
is not taken into account, the check will only be paid if the check is presented within….
a. 15 days from the date of issuance;
b. 30 days from the date of issuance;
c. 45 days from the date of issuance;
d. 60 days from the date of issuance; 46.
The typical risks in the collection order payment method are: a. Fake payment vouchers;
b. The document arrives that the payer's account has insufficient funds to pay;
c. Debtor delays in making payment orders; d. All sentences are correct; 47.
The main uses of payment cards are: a. Cash withdrawals;
b. Payment for goods and services; c. Loan; d. Payment to the bank; 48.
Non-cash payment services in banks include:
b. Including payment service via payment account and not through customer's payment account;
c. Payment can be applied to State Treasuries in the same area;
d. Including payment services are collection orders, payment orders, checks and cards; e. All right; lOMoARcPSD| 36667950
1. Choose the correct statement:
a. Account holders who are purchasers can use non-cash payment methods including security checks,
payment orders; letter of credit to pay for goods and services;
b. Merchant account holders can use non-cash payment methods including money orders, money orders and letters of credit;
c. Individuals can use cash withdrawal cards (ATMs) to pay for goods and services anywhere; d. All right;
2. According to current regulations, based on the amount used for payment, bank cards can be classified into types:
a. Debit card, credit card, ATM card;
b. ATM card, credit card, overdraft card;
e. ATM card, credit card, check payment security card; a.
Prepaid cards, debit cards, credit cards;
61. What is the difference between a credit card and a debit card? a. Payment uses of the card; b. Scope of card use; c. Card payment technology;
d. Origin of the amount on the card;
62. Which of the following statements does not describe the characteristics of the payment authorization method:
a. The payment order goes from the debtor;
b. Suitable for payment for public goods used first, pay later;
c. The rotation process is consistent with the principle of debt first - credit later;
d. There are advantages for sellers;
63. Which of the following measures is the most effective to develop a highly secure and reliable
information infrastructure in the banking financial services industry: a.
Expand the supply of products and services; b. Increase computing speed; c.
Expand sales with multiple supply channels; d. Add new product applications;
64. : A business A in Ho Chi Minh City wants to pay for goods to a supplier in Hanoi, DN A should
use the following means: a. Traveller's Cheques; b. Bank card;
c. Authorization for collection; d. Accreditative;
Sentences The similarities in check payment and payment order payment methods are:
a. Payment documents issued by the debtor;
b. The possibility of encountering fake vouchers;
c. Is transferred directly in the transaction; d. All sentences are correct;
Which of the following statements is not true about non-cash payment intermediaries:
a. Not using cash but numbers on the books;
b. Not allowed to open payment accounts in foreign currencies;
c. Individuals who do not have full civil act capacity are not allowed to open accounts for transactions;
d. Payment card is one of the non-cash payment methods; Question: Choose the correct statement:
a. Debit and credit cards are both payment cards;
b. Debit card and credit card are both forms of lending according to the bank's credit limit;
c. Banks rely on debit and credit cards to assess the creditworthiness of their customers;
d. Debit and credit cards are not allowed to withdraw money at ATMs; lOMoARcPSD| 36667950
Question: Choose the correct statement from the following statements about the use of checks in payment:
a. Banks and State Treasury may only sell blank checks to customers who have accounts at their units;
b. The beneficiary has the right to appeal in all cases where the check is rejected;
c. If the check is not paid, the assignor of the check is liable to the holder of the check;
d. Damaged checks must not be crossed out, but torn from the checkbook; Question: Choose the
incorrect statement from the following statements:
a. Payment services via customer's payment account, including: money transfer, collection and payment;
b. Payment services not through the customer's payment account, including:
money transfer, collection and payment;
c. Non-cash payment service means a payment service that must go through a payment account;
d. Means of non-cash payment are electronic or paper vouchers;
Which sentence is not the same between payment by payment order and collection order?
a. There are 2 forms of payment through payment intermediaries;
b. Orders made by account holders must be made on pre-printed forms provided by payment service providers;
c. Is a form of payment that the account holder makes an order, authorizes payment to payment service providers to make payment;
d. Only used to pay different values, different periodicals;
Question : Arrange the following sentences in the correct order of payment:
1. Buyer and seller have a relationship of buying and selling goods and/or services with each other;
2. The buyer's bank pays the seller through the seller's bank;
3. The buyer writes a payment order to request his/her bank to pay for the purchase to the seller;
4. The seller's bank notifies the seller that the buyer has paid;a. 1, 2, 3, 4; b. 1, 3, 4, 2; c. 1, 3, 2, 4; d. 1, 2, 4, 3;
Question : Which of the following statements is true?
a. Payment authorization made by the bank/service-providing organization;
b. The collection mandate is established by the beneficiary;
c. Payment authorization is often used to pay different values, different periods;
d. Collection mandates usually only pay for fixed amounts periodically;
Question: The form of payment is made between commercial banks in different systems and
locations, but each commercial bank has a deposit account at the other bank. a. Inter-bank payment;
b. Payment through the State Bank; c. Clearing;
d. Payment through correspondent banking relationship;
Question: Disadvantages of one-time payment via SBV a. Slow payment speed;
b. Only applicable to banks in the same system;
c. Signature samples must be registered at the lead bank and other member banks; d. High cost of use; lOMoARcPSD| 36667950
Question: Which of the following statements is incorrect?
a. Anonymous check is a check that does not bear the beneficiary's name; and faceless checks are
transferrable only by endorsement;
b. Nominal check is a check that specifies the beneficiary's name; identification checks are not
transferable without endorsement;
c. Wire checks are non-transferable and non-cashable;
d. Cash checks paid in cash by the clearing bank;
Question 2: Providing payment service via account is:
a. Provision of means of payment and credit granting;
b. Provision of payment means and financial leasing;
c. Provision of payment means and securities brokerage;
d. Provide payment means and execute payment orders;
Question : The disadvantages of payment by Collection Order are:
a. Cannot be used in the case of payment for services provided with an accurate means of measurement;
b. The collection order is made by the beneficiary, so there may be a delay in the payment process ;
c. Using the Collection Mandate takes a lot of time and costs;
d. Collection mandate is only used in transactions of goods/services of a public nature;
Question : The advantages of the form of clearing between banks are:
a. The payment range is wide, nationwide because the network of branches of the
State Banks covers all localities across the country;
b. If banks have to pay but the balance is not enough to pay, the State Bank and commercial banks will
lend to supplement the shortfall in clearing;
c. Can quickly process many payment transactions at the same time;
d. It is possible to process many transactions between banks that have payment relations at the same time;
Question : Which of the following is not a feature of a debit card?
a. Withdraw money directly from your account to make the payment;
b. Ability to withdraw money from ATMs;
c. Credit balance of card account decreases upon payment;
d. The debit balance of the account increases upon payment;
Question: Common characteristics of all payment methods between banks are: a. No cash;
b. Execute transactions one by one;
c. Performing multiple transactions at the same time;
d. Nationwide payment coverage;
Sentence Differences between the Czech Republic and the payment authorization: a. Payment order maker; b. Payment order recipient;
c. Time of handing over documents; d. Beneficiaries of vouchers;
Question: What is the risk in the payment method by collection order: a. Fake payment vouchers;
b. Insufficient funds in the payer's account when the documents arrive;
c. Debtor delays in making payment orders;
d. Creditors are late when making payment orders;
Which of the following statements is incorrect when talking about the utility of payment services via NH:
a. Reduce costs of transportation, storage, cash inventory; lOMoARcPSD| 36667950 b. Avoid all possible risks;
c. Effectively combat tax loss;
d. To be provided with banking services with many incentives;
Question Which of the following methods of collecting electricity service charges brings the most
benefits and advantages to a water company? a. Toll collection at home;
b. Collect fees through collection mandate;
c. Collect fees through payment authorization; d. Receive payments by check;
Question Which form of payment between banks has the most advantages when making
payments between banks in the same system: a. Inter-bank payment; b. Inter-bank payment; c. Payment via agent bank; d. Clearing;
Question Characteristics of clearing between banks are:
a. Can process multiple payment transactions at the same time; b. Slow payment speed;
c. Only applicable to banks in different systems;
d. The member banks are not required to open an account at the leading bank; Question What is the
BIN number on the ATM card?
a. Cardholder's personal identification number;
b. Code indicating the cardholder's transaction account; c. Card issuing bank's code;
d. The card's password is provided by the bank upon issuance;
Which of the following statements is not true about SWIFT:
a. The fast speed allows to process a large number of transactions in any country in the world;
b. Very high security and safety;
c. The cost for an electric transaction is lower than the traditional means of communication;
d. Using SWIFT will follow a uniform standard;
Question When there are multiple checks filed in the bank at the same time to collect money from
a drawer but the drawer's ability to pay is not enough to pay all of those checks, the order of payment will be:
a. In order of check number and draw date;
b. By date of draw and order of check number;
c. According to the value of the amount on each check;
d. NH will not pay all such checks;
Question Currently, among the non-cash payment means, which means of payment has the largest
total transaction volume in Vietnam?
a. Checks; b. Accreditative;
c. Authorization for collection; d. Bank card;
Which of the following statements about checks is true :
a. Past the validity period of the check, in order to receive payment, the beneficiary must request the drawer's bank to pay;
b. When a check has been ordered, the payee is responsible for paying the amount stated on the check
even if the time limit for presentation of the check has expired;
c. The buyer of the traveler's check that signs the check can convert it into cash equivalent to the amount
written on the check by the buyer;
d. For crossed checks, which are non-cashable checks; The advantage of collection mandate is:
a. Can be used for both domestic and international payments;
b. The bank can control the solvency of the payer; lOMoARcPSD| 36667950
c. The payment process is fast but complicated;
d. Reduce costs for banks;
Question Suppose you are a payment officer, when receiving a payment order from a customer,
what should you pay attention to first?
a. The beneficiary does not have an account at the bank;
b. The payer does not have an account at the bank;
c. Beneficiary does not have enough balance in the bank account;
d. The payer does not have enough balance on the bank account; Sentence
Which of the following is true about spending and revenue authorizations?
A. Payment authorization and collection authorization are both established by the bank
(Wrong, the payment order and the collection order will be made by the person who needs to pay or
collect money, who is not always the bank.)
B. When using a payment order and a collection order, the buyer and seller must have an account at the same bank
(Wrong, for a payment order, the buyer and seller do not necessarily have the same bank account.
For a collection order, the buyer and seller must both have a bank account (same or different bank) )
C. Remittance service fee to be paid by the payer or the beneficiary ( Yes, depending on the agreement,
the payer or the beneficiary will pay the bank transfer fee)
D. Beneficiary's bank receives remittance fee
(Wrong, the money transfer fee will be borne by the payer's bank) Sentence is used between:
a. Banks in the same system, same location, regular transactions;
b. Banks in the same system, different locations, infrequent transactions;
c. Banks are different from the system, in different locations, with regular transactions;
Banks are different from the system, in the same area, with irregular transactions; d.
Sentence In payment of checks, the person whose checking account is debited: a. Beneficiary; b. Drawer; c. Transferor; d. Payee;
Question Type of check guaranteed to be paid by the bank to the beneficiary by freezing
the check issuer's amount at the bank is: a. Security check ; b. Cash checks ; c. Check slash ; d. Check by order ;
Crossed check sentence :
a. Beneficiaries do not need to have an account at NH ;
b. The issuer does not need to have an account at NH ;
c. The issuer must have an account at the bank ;
d. The beneficiary must have an account at the bank ;
The sentence “…………………….. is an unconditional payment order, signed by a bank customer,
ordering the bank to deduct a certain amount of money from his account to pay. to the designated person”.
a. Promissory Note (Promissory Note) ;
b. Bill of Exchange (Bill of Exchange) ; c. Check (Check) ; d. Letter of Credit (L/C) ;
The sentence "Act of commitment to pay by the person obligated to pay when the bill of exchange
reaches maturity" is a transaction:
e. Acceptance of bill of exchange lOMoARcPSD| 36667950
f. Endorsement of bills of exchange
g. Guarantee of bills of exchange (Aval) h. Protest (Protest)
Question Account holders with payment deposits at banks can use:
a. Checks, credit cards, payment orders, collection orders;
b. Checks, ATM cards, payment orders, collection orders;
c. Checks, credit cards, ATM cards, payment orders;
d. Checks, promissory notes, credit cards, ATM cards;
Question If the time of force majeure event is not counted, the check will only be paid if the check is presented within:
a. 10 days from the date of issuance;
b. 20 days from the date of issuance;
c. 60 days from the date of issuance;
d. 30 days from the date of issuance;
When paying by payment order submitted by the customer to the bank, the payment staff should
pay attention to the following problems: a. The beneficiary does not have a bank account;
b. The payer does not have an account at the bank;
c. The beneficiary does not have enough balance on the account at the bank;
d. The payer does not have enough balance on the account at the bank;Sentences The
mode of payment for letters of credit is almost exclusively used in payment of
import-export contracts and is rarely used to pay for domestic goods sale and
purchase contracts because:
a. Domestic customers are limited in foreign languages;
b. The payment procedure is too complicated;
c. Customers have no need to use this method in payment of domestic sales contracts;
d. The importer is not fluent in the language of the exporter;Sentence Payment cards can be used
for: a. Any debt payments to the bank;
b. Any payment for services accepted by the beneficiary;
c. Any payment accepted by the beneficiary;
d. Any payment for goods accepted by the beneficiary;
Sentence Element not required when issuing bill of exchange a. Place of drawing of bills of exchange;
b. An unconditional order to demand money;
c. Amount in figures and words of the bill of exchange; d. Title of bill of exchange;
Which of the following statements is most accurate:
a. Payment via bank is a type of money receiving service, whereby the bank performs the receipt of
money from the account of the payer, by debiting the account, to the account of the beneficiary, by credit the account;
b. Payment via bank is a type of money receiving service, whereby the bank makes a deduction to
transfer money from the account of the payer, by debiting the account, to the account of the
beneficiary, by how to credit the account;
c. Payment via bank is a type of payment service whereby the bank makes a deduction to transfer money
from the account of the payer, by crediting the account, to the account of the beneficiary, by how to debit the account;
d. Payment via bank is a type of payment service whereby the bank makes a
deduction to transfer money from the account of the payer, by debiting the account, to the account of
the beneficiary, by how to credit the account; The mandator's sentence is:
a. Buyers and sellers of goods;
b. Buyers of goods and services;
c. Person with balance in a checking account;
d. Sellers of goods and services; lOMoARcPSD| 36667950
The difference between a debit card and a credit card is:
a. Credit cards are used to grant credit, debit cards are used to pay debts
b. Credit cards do not require customers to have money on their accounts to use,debit cards require
customers to have money on their accounts to use;
c. Credit cards issued by credit institutions, debit cards issued by banks:
d. Credit cards require customers to have money on their accounts to be used, debit cards do not
require customers to have money on their accounts to be used;
Sentence The difference in usage between blank endorsement and command endorsement is: a.
Endorsement blank for the purpose of transferring the bill of exchange by hand, endorsing
to order for the purpose of transferring the draft draft to the order of the beneficiary; b.
Endorsement is blank for the purpose of facilitating the transfer of bills of exchange,
endorsing by order is for controlling the transfer of drafts; c.
Endorsement is blank for the purpose of facilitating the transfer of drafts, endorsing by
order is for the purpose of terminating the transfer of drafts; d. The endorsement is blank for the
purpose of facilitating the transfer of the bill of exchange, the endorsement by order is for the
purpose of restricting the transfer of the bill of exchange;
The characteristics of the documentary credit payment method are stated as follows: a.
Documentary credit payment method, the importer commits to pay the exporter's bank; b.
Documentary credit payment method, the importer's bank commits to pay theexporter's bank; c.
Documentary credit payment method, the importer's bank commits to pay the exporter; d.
Documentary credit payment method, the importer commits to pay the exporter;
Sentences of the method of payment by collection with documents under D/P condition are stated as follows: a.
Payment method by collection with documents under D/P terms, the exporter cannot deliver
the goods without receiving the money from the importer; b.
Payment method by collection with documents under D/P terms, the exporter can receive
money without delivering the goods to the importer; c.
Payment method by collection with documents under D/P terms, importer cannot receive
goods without paying exporter; d.
Payment method by collection with documents under D/P terms, the exporter can receive the
goods without paying the exporter;
Sentences of the method of payment by collection with documents under D/A condition are stated as follows:
a. The exporter cannot deliver the goods without receiving payment from the importer;
b. The exporter can receive money without delivering the goods to the importer;
c. The importer cannot receive the goods without paying the exporter;
d. The importer can receive the goods without paying the exporter;
Sentence According to the current regulations of Vietnam, the time limit for presenting a request
for payment of a check is ....., from the date of issuance: a. 30 days; b. 60 days; c. 45 days; d. 6 months;
Question Payment of checks shall be terminated after ……., from the draw date indicated on the check: a. 30 days; b. 60 days; c. 45 days; d. 6 months;
If the check is presented after the time limit for presentation for lOMoARcPSD| 36667950
payment but not more than ………… from the date of drawing, the drawee can still pay if the
drawee does not receive the notice of suspension of payment for check and the drawer has
sufficient funds on the account to pay.
a. 30 days; b. 60 days; c. 45 days; d. 6 months;
Which of the following cards can be used by customers when their deposit accounts are exhausted: a. Debit card; b. Escrow card;
c. Card used by overdraft method; d. Credit;
Sentence In payment of a check, the drawee is:
a. The person who is obligated to pay;
b. The person serving the payer; c. Beneficiary of the check;
d. The person serving the payee of the check;
Which of the following means of payment is no longer used in Vietnam:
a. Bill of exchange for debt receipt;
b. Bill of exchange for debt collection; c. Payment check; d. Accreditative;
Which of the following states the difference between a check and a payment order:
a. Subjects make different payment orders;
b. The probability of counterfeit vouchers in checks is higher than in payment orders;
c. Checks are transferred directly in the transaction, while the payment order is not;
d. Sentences b and c are correct; Bank Card is used to: a. Cash withdrawals
b. Payment for goods and services c. Bank loan e. Both a and b are correct
Sentence Distinguish between magnetic cards and electronic cards in: b. Origin of the card;
c. The type of card reader suitable for it; d. Scope of card use;
e. Methods of immediate or deferred payment;
Question Based on the technology of the card, the bank's cards include: f. Magnetic and chip cards; g. Magnetic and credit cards;
h. Magnetic cards and ATM cards; i. Chip cards and ATM cards;
Question Payment by check will
f. Faster than paying by proxy; g. Slower than paying by ATM; lOMoARcPSD| 36667950
h. All forms of payment are the same;
i. Not enough grounds to conclude the above ideas;
Sentence The method of payment between customers via bank is a.
The Bank deducts from the payer's account to the beneficiary's account through the Bank's operations; b.
The Bank deducts from the bank's account and transfers it to the payer's account to pay the beneficiary; c.
The bank deducts from the beneficiary's account and transfers it to another bank related to the customer in payment; d. All of the above are true;
Question According to current regulations, the amount used for debit card payment is from the account: a. Term deposit account; b. Payment deposit account; c. Demand savings account; d. Current account;
Which of the following check types is not subject to a time limit for payment: a. Cash check b. Traveller's Cheques c. Security check d. Check slash
Which of the following statements does not describe the characteristics of the payment authorization method:
a. The payment order goes from the debtor;
b. Suitable for payment for public goods used first, pay later;
c. The rotation process is consistent with the principle of debt first - credit later;
d. There are advantages for sellers;
Which of the following characteristics best describes the nature of the collection mandate?
a. Used for goods transactions between two parties that have regular relations;
b. The payment order goes from the debtor;
c. The process of circulating documents is like a check;
d. Buyers have the advantage of being paid late;
Question When commercial banks provide good payment facilities, they will attract: a. Source of savings deposit
b. Source of non-transactional deposits
c. Source of transaction deposit d. All sources above
Question To use the means of payment via bank, customers must have:
a. Current account at the bank
b. Term savings account at the bank
c. Payment deposit account at the bank d. a and c are correct. Part 3
Which business sentence is an on-balance sheet transaction of NH? a. Buy corporate bonds; b. Advise; lOMoARcPSD| 36667950 c. Investment Management; d. Insurance;
Question 1. SMALL financial services
A. Increase the capital source of the bank;
B. Decreasing the bank's assets;
C. It both increases capital and reduces the bank's assets;
D. Does not change the capital and assets of the bank.
Question 2. What rights does the trustee have over the trust property? A. Possession;
B. Possession, use; C. Settlement;
D. Possessing, using, a part of the right to dispose.
Question 2. In January/N, enterprise A has 15,000 USD. Currently, enterprise A has a capital need
of 300 million VND for production and a debt of 10,000 USD to be paid at the beginning of
February/N. What should businesses do to both have money for production and can eliminate
exchange rate risk when paying debts in February/N (knowing that the USD exchange rate is trending up)?

A. Bank loan of 15 000 USD with floating interest rate for 1 month term;
B. Bank loan of 15 000 USD with interest rate of 1.7%/year for 1 month term;
C. Sign a foreign currency swap contract worth USD 15 000;
D. Deposit 15,000 USD in the bank with the interest rate of 0.25%/year for 1 month term.
Question 1. Choose FALSE when talking about derivative services:
A. Derivatives are a type of financial hedging;
B. Derivative services originate from potential risks in economic contract transactions;
C. Derivative services are always born before a contract transaction;
D. Derivatives services help clients to hedge and minimize financial risks in addition to profit-making speculation.
Question 2. What are the characteristics of financial services?
A. Revenue from fees, not using capital, on-balance sheet activities;
B. Revenues from fees and interests, capital use, off-balance sheet activities;C. Income from
interest, not using capital, on-balance sheet activities;
D. Income from fees, not using capital, off-balance sheet activities.
Question 1. Services of modern banks are: A. Accepting deposits; B. Pay; C. Commission; D. Preservation, consignment.
Question 2. Choose the wrong sentence:
A. Entrustment contracts are not revocable;
B. The trust maker may retain only part of the control over the assets;
C. Liquidation of assets under a will is a form of trust;
D. In Vietnam, individuals, banks and financial institutions are entitled to entrust capital to invest.
Question 1. Choose FALSE. Receive entrustment from individual customers including: A.
Liquidation of assets according to the will; B. Securities trading; C.
Operate individual trusts under contract; D. Property guardian.
Question 1. What is NOT a consulting service? A. Low level of risk; B.
Requires a team of banking consultants to be reputable, knowledgeable and professional; C.
Content can be investment advice, purchase and sale, consolidation, merger; lOMoARcPSD| 36667950 D.
The consultant is responsible for the decisions that the consultant makes. Question 2. Choose FALSE: A.
In the trust service, almost all rights on the property are transferred by the owner to the trustee,
retaining only a part of the control of the property; B.
In the representation service, the owner still retains the right over the property, only transferring to
the representative the establishment and performance of a
number of civil transactions within the scope of competence; C.
In practice, trust contracts are often revocable; D.
There are three main types of agency services: contract representation, asset management, and litigation.
Question 1: Which of the following is a type of bank guarantee issued by a credit institution to the
guarantor to ensure the obligation to return the advance payment received by the customer to the
guarantor under the signed contract:
A. Payment Refund Guarantee; B. Payment guarantee; C. Bid guarantee; D. Loan guarantee.
Verse 2: Choose the FALSE sentence. The guarantee contract or guarantee commitment of the
guarantor must be signed by: A. Legal representative;
B. Underwriting risk manager; C. Guarantee appraiser; D. Guaranteed person.
Question 1. Choose the TRUE statement about Trust Services:
A. In Vietnam, only commercial banks are allowed to perform entrustment services;
B. In trust services, the trustee (the trustee) does not have the right to possess, use and partially dispose of the property;
C. Credit institutions or foreign bank branches are authorized by individuals;
D. The entrustor must bear the risk when performing entrustment operations in the field related to banking activities.
Question 2. Banks are not authorized to entrust or receive entrustment in the field of business: A. Asset Management B. Insurance business; C. Banking activities;
D. Buying and selling real estate.
Verse 1 . Which of the following statements false:
A. Financial services include any banking and other financial services (except insurance);
B. When the bank performs financial services, it will not change either side of thebank's balance sheet;
C. Financial services help banks supplement profits and spread risks in business;
D. Financial services in banking are services associated with banking activities.Question 2. In what way
is the guarantee by the method of selling all or nothing different from the other guarantee methods?
A. Commitment to buy all issued securities regardless of whether they are fully distributed or not;
B. If the amount of securities sold reaches a ratio lower than the required rate, the entire issuance will be canceled;
C. If the issued securities are not sold out, the entire issuance will be canceled and the money sold will be owned by the underwriter;
D. If the issued amount of securities is not sold out, the entire issuance will be canceled and the money
sold will be returned to the buyer.
Question 1. Which financial services do commercial banks not need to establish or acquire
subsidiaries or affiliates? A. Underwriting securities; B. Consulting services; C. Insurance; lOMoARcPSD| 36667950 D. Financial leasing.
Question 2: Choose the TRUE statement:
A. In preservation and deposit services, the Bank still has to pay interest to customers if the preservation is money;
B. Futures contracts are still traded on the OTC market;
C. An options contract does not oblige its buyer to execute the committed transaction, but an option
premium is required in all cases.
D. Large-scale financial leasing companies are still allowed to contribute capital, buy shares, establish
subsidiaries and associate companies.
Question: The similarity between agency service and fiduciary service is:
a. The owner retains the rights on the property;
b. Complex work, depending on the customer;
c. High legality, strictly regulated by the provisions of law;
d. Can be terminated by simple procedures;
Question : Which is not a primary agency job:
a. Contract signing representative; b. Management representative; c. Legal representative;
d. Personal tax processing representative;
Question: Which of the following is not true about the service of preserving valuables:
a. Items sent can be gold, money, souvenir photos;
b. The Bank must return the consignment intact;
c. Preserved money and gold are charged interest;
d. The bank knows the details of the items deposited;
Question: Financial services bring revenue to banks from: a. Credit interest; b. Fee collection; c. Financial investment;
d. Mobilizing deposits and providing payment services;
Question 1: In a trust service, what rights do the trustee have over the trust assets:
a. Right of possession and right of use;
b. Right of control and right of disposition; c. Possession only:
d. Nearly all rights as an owner to property;
Question 2: What rights does the trustee have with respect to the trust property: a. Partial control; b. Part of the right to use;
c. Partial decision-making power;
d. Have all the rights of 1 owner to the property;
Question 3: According to Circular 04/2012 TT_NHNN, credit institutions, foreign bank branches are
not allowed to receive entrusted capital from any of the following entities: a. Other credit institutions; b. Overseas organizations; c. Individuals; d. Domestic enterprises;
Question 2: Choose the wrong sentence: The bank can perform the operations related to the treasury as follows:
a. Collecting and paying on behalf of;
b. Transfer money at home and abroad;
c. Currency exchange, money counting;
d. Receive entrustment from customers; lOMoARcPSD| 36667950
Question: Choose the wrong sentence:
a. Commercial banks are entrusted with legal entities;
b. Custody and deposit services are classic banking services;
c. Leasing safes, safes belonging to financial leasing services;
d. Bank guarantee is a form of credit extension;
Question: Which service belongs to the banking entrustment service: a. Cash management;
b. Identify an efficient capital structure;
c. Entrust to issue securities for enterprises;
d. Consulting in financial management;
Which of the following services is not part of treasury services of commercial banks :
a. Exchange money at the request of the customer ; b Liquidation
of assets according to the will ;
c.Cash collection at the customer's location ; d. Remittance abroad ;
Which of the following transactions is not part of treasury services?
a. Transfer money at home and abroad ;
b. Open a payment deposit account ; c. Exchange money ; d. Household collection ;
Which of the following banking operations only collect fees, not interest? a. Factoring ; b. Loans ; c. Buy government bonds ; d. Payment via bank ;
Question What is the method of underwriting of securities that the underwriters follow the
instructions of the issuer to sell all of the securities, otherwise, the entire issuance will be
canceled?
a. Guarantee with the highest commitment;
b. Guarantee with firm commitment;
c. Guarantee under the minimum - maximum method;
d. Guarantee in the form of either the sale of all or nothing;
Question A customer deposits gold and has to pay fees to the custodian bank. The relationship
between the bank and the customer is:
a. The relationship between the borrower and the lender;
b. The relationship between the consignee and the depositor;
The relationship between the lessor and the lessee of the safe; Owners and agents;
The service of cash payment from Customer's account to the address requested by Customer is: a. Payment service using UNT;
b. Recurring UNC billing service; c. Treasury services;
d. Account management services and safes; Trustee functions: a. Master asset management;
b. Manage assets for the benefit of beneficiaries; c. Preserving valuables; d. a and b;
Which of the following is not true about commercial deed?
a. Commercial deed is called commercial paper;
b. Usually issued by reputable large enterprises;
c. Enterprises that want to issue commercial certificates need collateral;
d. For businesses with a low credit rating that want to issue commercial certificates, lOMoARcPSD| 36667950 a bank guarantee is required;
Which statement is not true about agency services:
a. The relationship between the customer and the representative is not as close as in entrustment;
b. An authorized representative on behalf of the customer in each specific case;
c. The agent has more rights over the TS than the fiduciary service; d. All is incorrect;
Which of the following measures is the most effective to develop a highly secure and reliable
information infrastructure in the banking financial services industry: e.
Expand the supply of products and services; f. Increase computing speed; g.
Expand sales with multiple supply channels; h. Add new product applications;
Question What form does the money transfer service take? a. Investment profession;
b. Foreign currency trading operations; c. Entrustment business; d. Consulting profession;
In trust services, which party is the profit and loss business results calculated for: a. Bank; b. Client; c.
If there is a loss, the bank will bear, the profit will be enjoyed by the customer; d.
If there is a loss, the customer will bear it, and the bank will enjoy the profit; Question Banks provide
consulting, entrustment, financial investment, gold and silver trading services mainly for the following purposes:
a. Reduce operational risks; b. Capital mobilization; c. Increased liquidity; d. Diversify revenue sources;
A bill of exchange accepted by a bank is considered a highly secure instrument because: e.
The bank is the person responsible for securing the payment to the beneficiary; f.
Issued and guaranteed by the Government; g.
Low interest rate should be high safety; h.
Be committed by the importer to repay the debt;
Question: Collecting, paying for, transferring money, changing money... What is the service? a. Commission; b. Funds; c. Representative; d. Advise;
Which of the following is not true about Banker's Acceptance? a. Not transferable ;
b. For a bank accepting a bill of exchange, agreeing to seal the acceptance is considered a form of
signature credit for the customer who is the issuer ;
c. A bill of exchange issued by an enterprise ;
d. Be stamped "accepted" by the bank on the draft ;
Commercial banks are NOT authorized to entrust, receive entrustment, or agent in the field related to: a. Banking activities; b. Insurance Business;
c. Manage assets in accordance with regulations of the State Bank; d. Personal trust;
Question Is the consultant legally responsible for the decisions made by the consulted client? a. Take full responsibility; lOMoARcPSD| 36667950
b. Take part of the responsibility;
c. Responsibilities will be divided in two; d. No liability;
Question The team of consulting service department of commercial banks is usually:
a. Any employee working in the NH;
b. Professional, experienced and knowledgeable staff in the fields of consulting services;
c. Senior staff with good achievements in the bank's activities;
d. Elderly employees have worked for a long time and are trusted by everyone;
Question What is the positive impact of diversification of banking services on commercial banks?
a. Risk dispersion; promote the joint development of professions;
b. Increase profits while increasing service expansion costs;
c. Serving customers in a package method, improving quality for customers;
d. Open more branches, easily redirect business; Sentence Functions of trust: a. Embrace asset management
b. Manage assets for the benefit of beneficiaries c. Preserving valuables d. a and b
Which of the following is not true about the bank safe deposit box service?
a. The procedure is very quick;
b. Assets are guaranteed to be absolutely safe;
c. The cost of using this service is lower than renting a safe;
d. Assets stored in this service are profitable;
Which of the following is not a financial service: a. Counting money; b. Guardianship of property; c. Accreditative; d. Customer consulting;
Which of the following statements is incorrect about the utility of preserving valuables and
valuable papers at a bank:
a. Customers are not allowed to check the property during the preservation period to ensure safety and secrecy;
b. The Bank commits to keep confidential information about the customer's TS;
c. To terminate the rental contract of the locker ahead of time;
d. To authorize another person when there is a written authorization in accordance with the law;
Which of the following statements is false about the preservation of assets at a bank?
a. The bank must preserve the absolute safety of the customer's assets. The property must be kept intact
until it is returned to the customer;
b. The bank must be responsible for the legal ownership of the assets deposited
with the bank for preservation;
c. Customer is responsible for fully and timely payment of preservation fee to the bank in accordance with regulations;
d. If the property box is returned with its lock and seal intact, the customer must be
responsible for the entire quantity and quality of the property contained in the iron box;
Which of the following is a representative service of a bank: a. Payment of wages to the customer's employees;
b. Distribution of assets on the will;
c. Consulting on real estate purchase and sale; lOMoARcPSD| 36667950
d. Managing the offices of BC company;
Which of the following sentences is correct:
a. A trust is a trustee who has the right to hold and manage such assets in perpetuity since the death of the trustee;
b. The representative has the right to perform certain tasks if the client sends a letter of appointment;
c. For fiduciary services, the trustee has no right to dispose of the assets;
d. The representative has a part of the right to dispose of the client's property;
Question In addition to areas related to banking activities, commercial banks are entitled to
entrust, receive entrustment, and act as agents in related fields? a. Rental Property; b. Insurance Business;
c. Buying and selling real estate;
d. Securities investment;
Sentence Choose the correct sentence, the "open" preservation method is:
a. Open a savings account at the Bank.
b. The method has high profitability for customers.
c. The form of money or gold is not profitable during the deposit period. d. Preserve valuables.
Question Choose the correct answer:
a. Leasing safes to customers is an “open” method of storage in storage and deposit services.
b. The maintenance fee for one box of property for one year is 0.5% of the property's value. c.
The service of preservation and consignment was born to ensure the safety and secrecy of assets for customers.
d. When the customer receives back the asset box with the lock and seal intact outside the box but the
entire quantity and quality of the assets contained in the box is affected or changed, the bank must
be responsible. The essence of guarantee services in banks: a.
A special type of credit; b.
A service for the purpose of promoting the bank's brand; c.
The Bank is the third party confirming the contract between the two established transaction parties; d.
Reducing risks in banking business;
Why is it said that custodial and depository services are a classic type of banking service:
a. Because this is the service that brings the bank the most revenue in the services.
b. Because this is a service that appeared right from the time the bank was born in history.
c. Because customers often go to the bank to ask the bank to perform this service the most.
d. Because this is a service available in all banks.
Which of the following is not true for the benefit of customers when using the service of preserving valuables:
a. The bank's valuable assets are absolutely safe;
b. Saving space and costs of purchasing equipment to store assets and papers;
c. Periodically check the value of assets by the bank;
d. To benefit from assets during the time of the bank's preservation; Question: Why do customers use
internet banking system in your opinion?
a. Because customers save time in querying account information and records of transactions of the day;
b. As customers can make all unlimited payments quickly and conveniently;
c. Because customers can make loans right on the system;
d. Because customers can pay service bills within the allowable limit;
D: For the payment of service bills through internet banking, banks do not limit the amount for customers. lOMoARcPSD| 36667950
Which of the following are the direct benefits of guarantee services for banks:
a. Bring a large number of transactions;
b. Bring in a source of income;
c. Enhance the reputation and brand of the bank;
d. Easy implementation of customer policy;
Question Difference between investment trust and term savings:
a. When the contract expires, customers who want to continue to entrust their investment do not need
to go through the procedures again;
b. The interest rate on investment trust is higher than that of term savings;
c. Savings deposits have a higher risk than investment trusts;
d. There is no difference, just different names;
Question 1. SMALL financial services
E. Increase the capital of the bank;
F. Reducing the assets of the bank;
G. It both increases capital and reduces the bank's assets;
H. Does not change the capital and assets of the bank.
Question 2. What rights does the trustee have over the trust property? E. Possession;
F. Possession, use; G. Settlement;
H. Possessing, using, a part of the right to dispose.
Question 1. Choose the TRUE sentence when talking about arbitrage trading and arbitrage speculation: A.
These two forms are essentially one because they are used to enjoy exchange rate differences; B.
These two forms are subject to exchange rate risk and require a certain amount of capital; C.
The administrator can determine the profit level due to the exchange rate difference from these two forms; D.
Currency arbitrage takes place at the same time while arbitrage speculation takes place at two different times.
Question 2. In January/N, enterprise A has 15,000 USD. Currently, enterprise A has a capital need
of 300 million VND for production and a debt of 10,000 USD to be paid at the beginning of
February/N. What should businesses do to both have money for production and can eliminate
exchange rate risk when paying debts in February/N (knowing that the USD exchange rate is trending up)?

A. Bank loan of 15 000 USD with floating interest rate for 1 month term;
B. Bank loan of 15 000 USD with interest rate of 1.7%/year for 1 month term;
C. Sign a foreign currency swap contract worth USD 15 000;
D. Deposit 15,000 USD in the bank with the interest rate of 0.25%/year for 1 month term.
Question 1. Choose FALSE when talking about derivative services:
A. Derivatives are a type of financial hedging;
B. Derivative services originate from potential risks in economic contract transactions;
C. Derivative services are always born before a contract transaction;
D. Derivatives services help clients to hedge and minimize financial risks in addition to profit-making speculation.
Question 2. What are the characteristics of financial services?
A. Revenue from fees, not using capital, on-balance sheet activities;
B. Revenues from fees and interests, capital use, off-balance sheet activities;C. Income from
interest, not using capital, on-balance sheet activities; lOMoARcPSD| 36667950
D. Income from fees, not using capital, off-balance sheet activities. Answer: EASY
Explanation : Financial services provide banks with revenue from fees, not from interest, like credit
or investment. In most financial services activities, banks often do not use capital, but only take
advantage of available facilities and advantages from experience. Because it is an off-balance sheet
activity, when the bank performs these activities, it does not change the two sides of the bank's balance sheet. LE TRIEU VU MSSV: 030326100308
Question 1. Services of modern banks are: E. Accepting deposits; F. Pay; G. Commission; H. Preservation, consignment. Answer:
Explanation: The services of deposit receipt, payment and preservation and deposit are traditional
banking services, which have existed for a long time. Trust service is one of the services of a modern
bank, playing an increasingly important role and generating more profits for the bank. Question 2.
Choose the wrong sentence:

E. Entrustment contracts are not revocable;
F. The trust maker may retain only part of the control over the assets;
G. Liquidation of assets under a will is a form of trust;
H. In Vietnam, individuals, banks and financial institutions are entitled to entrust capital to invest. Answer: EASY question Explain:
- According to Circular 04/2012/TT-NHNN on the profession of trusteeship and trusteeship of organizations NGUYEN NGOC THAI NGUYEN MSSV: 030326100151
Question 1. Choose FALSE. Receive entrustment from individual customers including:
E. Liquidation of assets according to the will; F. Securities trading;
G. Operate individual trusts under contract; H. Property guardian. Answer: REMOVE
Question 2. Choose FALSE . Financial leasing activities must meet one of the following conditions:
A. At the end of the lease term under the contract, the lessee is entitled to transfer the ownership of the
leased asset or continue to lease it as agreed by the two parties;
B. At the end of the lease term under the contract, the lessee has the right to preemptively purchase the
leased asset at a nominal price lower than the actual value of the leased asset at the time of acquisition;
C. The lease term of an asset must be at least 60% of the time required to depreciate the leased asset;
D. The total rental amount for an asset specified in the finance lease contract must be at least equal to
the value of that asset at the time of receiving the leased asset. Answer: EASY
Explanation : The total rent for an asset specified in a finance lease contract must be at least equal to
the value of that asset at the time of signing the contract. (Article 133 of the Law on Credit Institutions). DIEP THANH DANH MSSV: 030326100039 lOMoARcPSD| 36667950
Question 1. What is NOT a consulting service? E. Low level of risk;
F. Requires a team of banking consultants to be reputable, knowledgeable and professional;
G. Content can be investment advice, purchase and sale, consolidation, merger;
H. The consultant is responsible for the decisions that the consultant makes. Answer: EASY
Explanation : Legally, the consultant is not responsible for the decisions made by the consultant.
Because this service only provides information and best advice on the areas that customers are
interested in, they need to learn to help customers make accurate, effective decisions and minimize
risks. And the final decision still belongs to the customer and they are the ones who make that
decision, so they are responsible for the results.
Question 2. Choose FALSE:
E. In the trust service, almost all rights on the property are transferred by the owner to the trustee,
retaining only a part of the control of the property;
F. In the representation service, the owner still retains the right over the property, only transferring to
the representative the establishment and performance of a
number of civil transactions within the scope of competence;
G. In practice, trust contracts are often revocable;
H. There are three main types of agency services: contract representation, asset management, and litigation. Answer:
Explanation : Trust has a very high legal status, which is strictly governed by
legal regulations. The trustee holds the right to property, possession of the property has a legal
basis, the trustee must comply with the terms of the trust contract or court decisions. VUONG HOANG ANH : 030326100018
Question 1: Which of the following is a type of bank guarantee issued by a credit institution to the
guarantor to ensure the obligation to return the advance payment received by the customer to the
guarantor under the signed contract:
E. Payment Refund Guarantee; F. Payment guarantee; G. Bid guarantee; H. Loan guarantee.
Verse 2: Choose the FALSE sentence. The guarantee contract or guarantee
commitment of the guarantor must be signed by: E. Legal representative; F. Underwriting risk manager; G. Guarantee appraiser; H. Guaranteed person. Answer: EASY
According to Clause 1, Article 15, “Circular on regulations on bank guarantees” No. 28/2012/TT-NHNN lOMoARcPSD| 36667950
Question 1. Choose the TRUE statement about Trust Services:
E. In Vietnam, only commercial banks are allowed to perform entrustment services;
F. In trust services, the trustee (the trustee) does not have the right to possess, use and partially dispose of the property;
G. Credit institutions or foreign bank branches are authorized by individuals;
H. The entrustor must bear the risk when performing entrustment operations in the field related to banking activities.
Question 2. Banks are not authorized to entrust or receive entrustment in the field of business: A. Asset Management B. Insurance business; C. Banking activities;
D. Buying and selling real estate. Answer: EASY
Explanation: The Law on Credit Institutions, Article 106 only states that “Commercial banks are
entitled to entrust, receive entrustment, and act as agents in the fields related to banking activities,
insurance business, and asset management according to regulations. regulations of the State Bank”.
Verse 1 . Which of the following statements false:
a. Financial services include all banking and other financial services (exceptinsurance);
b. When the bank performs financial services, it will not change either side of the bank's balance sheet;
c. Financial services help banks supplement profits and spread risks in business;
d. Financial services in banks are services associated with banking activities.
Question 2. In what way is the guarantee by the method of selling all or nothing different from the
other guarantee methods?
a. Commitment to buy all issued securities regardless of whether they are fullydistributed or not;
b. If the amount of securities sold reaches a ratio lower than the required rate, theentire issuance will be canceled;
c. If the issued securities are not sold out, the entire issuance will be canceled andthe money sold will
be owned by the underwriting organization;
d. If the issued amount of securities is not sold out, the entire issuance will be canceled and the money
sold will be returned to the buyer.
Question 1. Which financial services do commercial banks not need to establish or acquire
subsidiaries or affiliates? a. Underwriting securities; b. Consulting services; c. Insurance; d. Financial leasing.
Question 2: Choose the TRUE statement:
a. In preservation and deposit services, the Bank still has to pay interest to customers if the preservation is money;
b. Futures contracts are still traded on the OTC market;
c. An options contract does not oblige its buyer to execute the committed transaction, but an option
premium is required in all cases.
d. Large-scale financial leasing companies are still allowed to contribute capital, buy shares, establish
subsidiaries and associate companies.